You are on page 1of 114

Pediatric orthopaedic

Self-Assessment Examination 2013

ANSWER Book
All rights reserved. No part of Pediatric Orthopaedic Self-Assessment Examination may be reproduced,
stored in a retrieval system, or transmitted in any form or by any means (electronic, mechanical,
photocopying, recording, or otherwise) without the prior written permission of the publisher.

Published January 2013


American Academy of Orthopaedic Surgeons
6300 North River Road
Rosemont, IL 60018

Copyright 2013 by the American Academy of Orthopaedic Surgeons

Requests for permission to reproduce any part of the work should be mailed to:
Attention: Examinations Department
American Academy of Orthopaedic Surgeons
6300 North River Road
Rosemont, IL 60018

Printed in the USA

2013 American Academy of Orthopaedic Surgeons 2013 Pediatric Orthopaedic Self-Assessment Examination

2013 American Academy of Orthopaedic Surgeons 2013 Pediatric Orthopaedic Self-Assessment Examination
Dear Colleague:

Thank you for purchasing the 2013 Pediatric Orthopaedic Special Interest Examination. To help
you obtain maximum benefit from this examination, I would like to briefly discuss its rationale
and some of its new features.

First, this examination is designed for use by orthopaedic surgeons in clinical practice. We have
made it a priority to present clinically relevant questions and avoid arcane orthopaedic facts.
Many of us are engaged in the American Board of Orthopaedic Surgery Maintenance of
Certification. This examination is also designed to help fulfill requirements or prepare for this
ongoing process.

Second, you will notice that this examination focuses on four areas: spine, trauma, hip, and
foot/ankle (although it covers many other important pediatric orthopaedic conditions). This is by
design. These areas have evolved rapidly during the last several years, and this examination
strives to reflect current knowledge in these fields. I hope that this examination will update
orthopaedic surgeons knowledge of these fields and stimulate further learning about these
topics.

Third, you will encounter new types of questions in this booklet. Many questions in this
examination are presented in the classic format that tests recall, interpretation of tests and
imaging, and problem-solving ability. This examination also introduces new formats such as
extended matching and multiple questions based on a single clinical scenario. These formats
allow for a broader, more inclusive examination and are designed to accurately assess a
participants clinical knowledge. We have incorporated advanced imaging such as MRI scans
and video into the examination as well.

Thank you again for choosing the 2013 Pediatric Orthopaedic Special Interest Examination. On
behalf of the Pediatric Evaluation Committee and the AAOS examinations staff, it is my hope
that you will regard this examination as a valuable program in your ongoing professional
education.

Sincerely,

B. David Horn, MD
Chair, Pediatric Evaluation Committee

2013 American Academy of Orthopaedic Surgeons 2013 Pediatric Orthopaedic Self-Assessment Examination
2013 American Academy of Orthopaedic Surgeons 2013 Pediatric Orthopaedic Self-Assessment Examination
ACCREDITATION
The American Academy of Orthopaedic Surgeons is accredited by the Accreditation Council for
Continuing Medical Education to provide continuing medical education for physicians.

AMA PRA CREDIT


U. S. Physicians: The American Academy of Orthopaedic Surgeons designates this enduring
material for a maximum of 12 AMA PRA Category 1 Credits. Physicians should claim only
the credit commensurate with the extent of their participation in the activity.

International Physicians: AMA PRA credit may only be claimed by, and awarded to,
physicians, defined by the AMA as individuals who have completed an allopathic (MD),
osteopathic (DO), or an equivalent medical degree from another country.

Allied Health Professionals: The American Academy of Orthopaedic Surgeons is not accredited to
offer credit for nurses and other allied health professionals. To determine if activities offering AMA PRA
Category 1 Credits are acceptable for your licensing or certification needs, please contact the relevant
organization(s) directly.

Independent Study (previously called Self-Scored) Examination Participants: The AAOS no


longer awards CME credit for printed independent study self-assessment examinations as a result
of new requirements from the AMAs Physician Recognition Award Program.

EDUCATIONAL OBJECTIVES
As a result of taking the 2013 Pediatric Orthopaedic Self-Assessment Examination, I am able to
describe the diagnosis and treatment of common pediatric foot and ankle conditions.
recognize, evaluate and treat pediatric trauma patients.
discuss the etiology and treatment of neurologic and developmental pediatric orthopaedic
conditions.
formulate a diagnosis and treatment plan for pediatric spinal disorders.
describe the diagnosis and treatment of pediatric hip disease.

INQUIRIES
Questions regarding completing this CME activity or other comments may be sent to
exams@aaos.org or write to: Attention: Examinations Department, American Academy of
Orthopaedic Surgeons, 6300 North River Road, Rosemont, IL 60018.

RELEASE DATE: January 2013


EXPIRATION DATE: December 31, 2015.
No CME credit will be awarded for this activity after December 31, 2015

2013 American Academy of Orthopaedic Surgeons 2013 Pediatric Orthopaedic Self-Assessment Examination

2013 American Academy of Orthopaedic Surgeons 2013 Pediatric Orthopaedic Self-Assessment Examination
Produced by the American Academy of Orthopaedic Surgeons
Pediatric Evaluation Committee of the Central Evaluation Committee

B. David Horn, MD, Editor in Chief


Assistant Professor, Clinical Orthopaedic Surgery, Perelman School of Medicine,
University of Pennsylvania, Philadelphia, Pennsylvania

Contributors
Matthew J. Bueche, MD
Pediatric Orthopaedic Surgeon, Private Practice, M & M Orthopaedics, Naperville, Illinois
John J. Grayhack, MD
Associate Professor of Orthopaedic Surgery, Northwestern University Feinberg School of
Medicine, Children's Memorial Hospital; Director of Resident Education, Division of
Orthopaedic Surgery, Childrens Memorial Hospital, Chicago, Illinois
William L. Hennrikus Jr, MD
Professor of Orthopaedics and Rehabilitation, Associate Dean of Education, Medical
Director Pediatric Bone and Joint Center, Penn State College of Medicine, Hershey,
Pennsylvania
Harish S. Hosalkar, MD
Attending Orthopaedic Surgeon, Co-Director International Hip Center, Director Hip
Research Program, Rady Children's Hospital, University of California San Diego, San
Diego, California
Eric T. Jones, MD
Associate Professor of Orthopaedic Surgery, Wayne State University; Pediatric
Orthopaedic Surgeon, Children's Hospital of Michigan, Detroit, Michigan
Kathleen A. McHale, MD
Professor of Surgery, Uniformed Services University for the Health Sciences, Clinical
Professor of Orthopaedic Surgery, George Washington University, Washington, DC
Kevin M. Neal, MD
Assistant Professor of Orthopaedic Surgery, University of Florida College of Medicine;
Instructor of Orthopaedic Surgery, Mayo Clinic College of Medicine, Jacksonville, Florida
Juan A. Realyvasquez, MD
Associate Professor of Orthopedics and Pediatrics, Drexel University College of Medicine
Orthopedic Center for Children, St. Christopher's Hospital for Children, Philadelphia,
Pennsylvania
Brian G. Smith, MD
Director of Pediatric Orthopaedics, Yale New Haven Children's Hospital; Associate
Professor, Associate Program Director, Department of Orthopaedics, Yale University
School of Medicine, New Haven, Connecticut
Samantha A. Spencer, MD
Instructor, Department of Orthopaedic Surgery, Harvard Medical School; Orthopaedic
Surgeon, Boston Childrens Hospital, Boston, Massachusetts
Walid K. Yassir, MD
Associate Chief, Pediatric Orthopaedics, Childrens Hospital of Michigan, Detroit,
Michigan

2013 American Academy of Orthopaedic Surgeons 2013 Pediatric Orthopaedic Self-Assessment Examination

2013 American Academy of Orthopaedic Surgeons 2013 Pediatric Orthopaedic Self-Assessment Examination
ACKNOWLEDGMENTS

Medical Editing
Brenda Moss Feinberg, ELS, AAOS Examinations Consultant

Graphic Design and Digital Media Specialist


John Cisco, AAOS Examinations Consultant

American Academy of Orthopaedic Surgeons


Constance M. Filling, Chief Education Officer

Department of Electronic Media, Examinations, CME Course Operations, and Practice Management
Howard Mevis, Director
Laura Hruska, M. Ed., Manager, Examinations
Marcie L. Lampert, Senior Coordinator, Examinations
Anna M. Scheer, Senior Coordinator, Examinations
Denise Plasky, Assistant, Examinations
Irene Bogdal, Administrative Assistant, Examinations

2013 American Academy of Orthopaedic Surgeons 2013 Pediatric Orthopaedic Self-Assessment Examination

2013 American Academy of Orthopaedic Surgeons 2013 Pediatric Orthopaedic Self-Assessment Examination
Disclosure Information for the 2013 Pediatric Orthopaedic Self-Assessment Examination

Bernard David Horn, MD (Chair): 4 (Johnson & Johnson);7 (JayPee Brothers medical
Publishing Comapny);9 (AAOS); Submitted on: 04/05/2012. *
Matthew J. Bueche, MD (Member): 9 (Pediatric Orthopaedic Society of North America);
Submitted on: 03/22/2012. *
John J. Grayhack, MD (Member): 4 (DePuy, A Johnson & Johnson Company; Medtronic
Sofamor Danek);9 (Scoliosis Research Society); Submitted on: 02/24/2012. *
William L. Hennrikus Jr, MD (Member): 9 (Pediatric Orthopaedic Society of North
America; Society of Military Orthopaedic Surgeons); Submitted on: 06/19/2012. *
Harish S. Hosalkar, MD (Member): 2 (Synthes); 3B (Allergan; Synthes); 4
(GlaxoSmithKline; Johnson & Johnson; Pfizer); 7 (Journal of Bone and Joint Surgery -
American); Submitted on: 04/30/2012. *
Kathleen A. McHale, MD (Member): 9 (AAOS); Submitted on: 05/21/2012. *
Kevin M. Neal, MD (Member): 1 (Orthopediatrics); Submitted on: 04/03/2012. *
Juan A. Realyvasquez, MD (Member): (n) Submitted on: 03/23/2012. *
Brian G. Smith, MD (Member): 2 (Stryker);3B (Stryker);5 (Stryker);9 (Section on
Orthopaedics, American Academy of Pediatrics Pediatric Orthopaedic Society of North
America Scoliosis Research Society); Submitted on: 04/17/2012. *
Samantha A. Spencer, MD (Member): 9 (AAOS Massachusetts Orthopaedic Association
Pediatric Orthopaedic Society of North America); Submitted on: 02/09/2012. *
Walid K. Yassir, MD (Member): (n) Submitted on: 04/20/2012. *

Contributors
Eric T Jones, MD (Detroit, MI): 3B (Globus Medical);8 (Journal of Pediatric Orthopedics;
Journal of Bone and Joint Surgery - American); Submitted on: 04/20/2012. *

American Academy of Orthopaedic Surgeons Staff


Howard Mevis (Rosemont, IL): 4 (GE Healthcare; 3M; Novartis);9 (Orthopaedic Learning
Center); Submitted on: 09/05/2012. *
Laura Hruska (Rosemont, IL): (n) Submitted on: 04/30/2012. *
Marcie Lampert (Staff Liaison): (n) Submitted on: 09/07/2012. *

* Disclosure Items Answered: (n) = Respondent answered 'No' to all items indicating no conflicts.
1= Royalties from a company or supplier; 2= Speakers bureau/paid presentations for a company or supplier; 3A=
Paid employee for a company or supplier; 3B= Paid consultant for a company or supplier; 3C= Unpaid consultant
for a company or supplier; 4= Stock or stock options in a company or supplier; 5= Research support from a company
or supplier as a PI; 6= Other financial or material support from a company or supplier; 7= Royalties, financial or
material support from publishers; 8= Medical/Orthopaedic publications editorial/governing board; 9= Board
member/committee appointments for a society.

2013 American Academy of Orthopaedic Surgeons 2013 Pediatric Orthopaedic Self-Assessment Examination

2013 American Academy of Orthopaedic Surgeons 2013 Pediatric Orthopaedic Self-Assessment Examination
Disclaimer
The material presented in this self-assessment examination has been made available by the
American Academy of Orthopaedic Surgeons for educational purposes only. This material is not
intended to present the only, or necessarily the best, methods or procedures for the medical
situations discussed, but rather is intended to represent an approach, view, statement, or opinion
of the author(s) or producer(s), that may be helpful to others who face similar situations.

Some drugs or medical devices demonstrated in Academy courses or described in Academy print
or electronic publications have not been cleared by the Food and Drug Administration (FDA) or
have been cleared for specific uses only. The FDA has stated that it is the responsibility of the
physician to determine the FDA clearance status of each drug or device he or she wishes to use in
clinical practice.

Furthermore, any statements about commercial products are solely the opinion(s) of the author(s)
and do not represent an Academy endorsement or evaluation of these products. These statements
may not be used in advertising or for any commercial purpose.

2013 American Academy of Orthopaedic Surgeons 2013 Pediatric Orthopaedic Self-Assessment Examination

2013 American Academy of Orthopaedic Surgeons 2013 Pediatric Orthopaedic Self-Assessment Examination
2013 Pediatric Orthopaedic Self-Assessment Examination Answer Book 11

Figure 1

Question 1
Figure 1 is the radiograph of a child who has pain in his left thigh. Extensive pigmentation with a jagged
border is seen on the left trunk. This child is at increased risk for

1. patellar instability and nail abnormalities.


2. involvement of other bones and precocious puberty.
3. cardiac abnormalities and hearing loss.
4. syndactyly and fibular hypoplasia.

PREFERRED RESPONSE: 2

DISCUSSION
The radiograph shows coxa vara with a lucency in the femoral neck, typical of the ground glass
appearance of fibrous dysplasia. Hyperpigmentantion with a Coast of Maine border is indicative of
McCune-Albright syndrome, which consists of polyostotic fibrous dyplasia and endocrine disturbances,
especially precocious puberty. Nail deformity, patellar instability, cardiac abnormalities, hearing loss,
syndactyly, and fibula abnormalities are not associated with this syndrome.

RECOMMENDED READINGS
Vlkl TM, Drr HG. McCune-Albright syndrome: clinical picture and natural history in children and
adolescents. J Pediatr Endocrinol Metab. 2006 May;19 Suppl 2:551-9. Review. PubMed PMID: 16789617.
Parekh SG, Donthineni-Rao R, Ricchetti E, Lackman RD. Fibrous dysplasia. J Am Acad Orthop Surg.
2004 Sep-Oct;12(5):305-13. Review. PubMed PMID: 15469225.
McCune-Albright syndrome. Polyostotic fibrous dysplasia. A.D.A.M. Medical Encyclopedia. http://www.
ncbi.nlm.nih.gov/pubmedhealth/PMH0002197 (Accessed 8/7/2012).

2013 American Academy of Orthopaedic Surgeons 2013 Pediatric Orthopaedic Self-Assessment Examination
12 American Academy of Orthopaedic Surgeons

Question 2
What gene is implicated in spinal muscular atrophy?

1. Survival motor neuron I (SMN-I)


2. Peripheral myelin protein 22 (PMP22)
3. Dystrophin
4. Androgen receptor

PREFERRED RESPONSE: 1

DISCUSSION
Deletions in the SMN-I gene are found in 95% to 98% of patients with spinal muscular atrophy. Genetic
testing is typically part of the diagnostic workup for spinal muscular atrophy. A positive test result is
diagnostic, and, in most cases, eliminates the need for muscle biopsy. The other choices are not associated
with spinal muscular atrophy. Defects in PMP22 are the cause of 70% to 80% of cases of Charcot-Marie-
Tooth disease. Mutations in the dystrophin gene cause Duchenne muscular dystrophy, and mutations in
the androgen receptor cause spinobulbar muscular atrophy (Kennedys disease).

RECOMMENDED READINGS
Sucato DJ. Spine deformity in spinal muscular atrophy. J Bone Joint Surg Am. 2007 Feb;89 Suppl 1:148-
54. Review. Erratum in: J Bone Joint Surg Am. 2007 May;89(5):1090-1. PubMed PMID: 17272431.
Sussman MD. Progressive neuromuscular diseases. In: Abel MF, ed. Orthopaedic Knowledge Update:
Pediatrics 3. Rosemont, IL: American Academy of Orthopaedic Surgeons; 2006:123-135.

2013 American Academy of Orthopaedic Surgeons 2013 Pediatric OrthopaedicSelf-Assessment Examination


2013 Pediatric Orthopaedic Self-Assessment Examination Answer Book 13

Video 3
*Reproduced from Caskey PM: Hip instability: Birth to six months. Orthopaedic Knowledge Online Journal 2006;4(1): http://orthoportal.aaos.org/oko/article.
aspx?article=OKO_PED010. Accessed September 14, 2012.

Question 3
Video 3 shows the physical examination of the right hip of a 14-day-old full-term female infant. There is
no history of breech presentation, she was born vaginally, and is the second child born to her mother. She
is otherwise healthy and has been eating and gaining weight appropriately. What is the most appropriate
course of action at this time?

1. Observation, with a repeat clinical examination in 2 weeks


2. Application of a hip abduction device and early follow-up to confirm reduction
3. Arthrogram and closed reduction of the right hip under general anesthetic, followed by spica
casting
4. Open reduction of the right hip with a capsulorrhaphy

PREFERRED RESPONSE: 2

DISCUSSION
The video shows an unstable right hip in a female infant. The visual clunk of the positive Barlow and
Ortolani tests is readily evident. Initial treatment in a young infant consists of application of an abduction
device to achieve and maintain hip location. Confirmation of successful hip location is usually performed
by ultrasound examination. In the United States, the Pavlik harness remains the most commonly used hip
abduction device, although others, such as hip abduction braces and the Frejka pillow, are available and
have been shown to have similar success. Observation is not necessary when the examination shows clear
instability. Arthrogram, closed manipulation, and spica casting are reserved for an older age group (those
between 6 and 18 months of age). Open reduction is appropriate if the hip remains dislocated after 18
months of age.

RECOMMENDED READINGS
Guille JT, Pizzutillo PD, MacEwen GD. Development dysplasia of the hip from birth to six months. J Am
Acad Orthop Surg. 2000 Jul-Aug;8(4):232-42. Review. PubMed PMID: 10951112.
Weinstein SL, Mubarak SJ, Wenger DR. Developmental hip dysplasia and dislocation: Part I. Instr Course
Lect. 2004;53:523-30. Review. PubMed PMID: 15116641.

2013 American Academy of Orthopaedic Surgeons 2013 Pediatric Orthopaedic Self-Assessment Examination
14 American Academy of Orthopaedic Surgeons

Figure 4a Figure 4b

Question 4
Figures 4a and 4b are the radiographs of a 7-year-old girl who fell and sustained an injury to her left elbow
in a foreign country 2 years ago. She was treated by a local healer with arm massage. Her pain and
swelling improved, but her mother noticed an abnormal appearance of the elbow that clinically manifested
as cubitus valgus. The child denied significant pain or functional limitations. Her neurologic and vascular
examination findings are normal. She denies fever, weight loss, or systemic illness. Her elbow is not
tender, there is no instability, and her range of motion reveals full flexion, extension, pronation, and
supination of the elbow. What is the most appropriate course of action?

1. Observation
2. Administration of intravenous antibiotics
3. Operative repair and bone grafting of the nonunited fragment
4. Distal humerus osteotomy to correct cubitus valgus

PREFERRED RESPONSE: 1

DISCUSSION
The patient and her parents should be counseled that although progressive cubitus valgus, pain, instability,
and tardy ulnar nerve palsy can occur, their incidence in this setting is unknown, and, as long as the girl
remains asymptomatic, observation is appropriate. The radiographs reveal a chronic nonunion of a lateral
humeral condyle fracture. Although the appearance could be confused with osteomyelitis, the patient
has a history of trauma and no clinical symptoms of infection. Fixation of the distal fragment has been
recommended to relieve pain and instability. Distal humerus osteotomy has been recommended to correct
cubitus valgus and to treat tardy ulnar nerve palsy.

RECOMMENDED READINGS
Toh S, Tsubo K, Nishikawa S, Inoue S, Nakamura R, Harata S. Long-standing nonunion of fractures of the
lateral humeral condyle. J Bone Joint Surg Am. 2002 Apr;84-A(4):593-8. PubMed PMID: 11940620.
Shimada K, Masada K, Tada K, Yamamoto T. Osteosynthesis for the treatment of non-union of the lateral
humeral condyle in children. J Bone Joint Surg Am. 1997 Feb;79(2):234-40. PubMed PMID: 9052545.

2013 American Academy of Orthopaedic Surgeons 2013 Pediatric OrthopaedicSelf-Assessment Examination


2013 Pediatric Orthopaedic Self-Assessment Examination Answer Book 15

Figure 5

Question 5
Figure 5 is the anteroposterior radiograph of a 7-year-old girl with a congenital limb difference. This type
of limb difference commonly is associated with

1. insufficient medial collateral ligament of the knee (MCL).


2. insufficient extensor mechanism.
3. absent lateral rays of the foot.
4. cleft hands.

PREFERRED RESPONSE: 3

DISCUSSION
Fibular hemimelia is associated with missing or absent lateral rays/toes, partial or complete absence of
the fibula, and anterior cruciate ligament insufficiency. Tibial hemimelia is associated with insufficient
extensor mechanism, clubfoot deformity, and, in some cases, cleft hands. MCL is not usually involved in
this disorder.

RECOMMENDED READINGS
Moseley C. Leg-length discrepancy. In: Morrissy RT, Weinstein SL, eds. Lovell and Winters Pediatric
Orthopaedics. 6th ed. Philadelphia, PA: Lippincott Williams & Wilkins; 2006:1213-1256.
Morissy RT, Giavedoni BJ, Couter-OBerry C. The child with a limb deficiency. In: Morrissy RT,
Weinstein SL, eds. Lovell and Winters Pediatric Orthopaedics. 6th ed. Philadelphia, PA: Lippincott
Williams & Wilkins; 2006:1329-1382.
Spencer SA, Widmann RF. Limb-length discrepancy and limb lengthening. In: Song KM, ed. Orthopaedic
Knowledge Update: Pediatrics 4. Rosemont, IL: American Academy of Orthopaedic Surgeons; 2011:219-
232.

2013 American Academy of Orthopaedic Surgeons 2013 Pediatric Orthopaedic Self-Assessment Examination
16 American Academy of Orthopaedic Surgeons

Figure 6a Figure 6b

Figure 6c Figure 6d

Question 6
Figures 6a through 6d are the CT scans of a 5-year-old girl who fell out of bed 4 weeks ago. She had
been treated for a nondisplaced clavicle fracture by her pediatrician and was referred for evaluation of
acute torticollis that was noticed after her fall. On examination, she had a fixed torticollis and resisted all
attempts at manipulation. The family denied a prior history of torticollis. What is the most appropriate
next step in her treatment?

1. Cervical traction
2. Sternocleidomastoid release
3. Use of a soft cervical collar
4. Posterior cervical fusion

PREFERRED RESPONSE: 1

DISCUSSION
Cervical traction, usually initiated with a Halter device, is the most appropriate initial treatment. Halo
traction may eventually be required, especially in long-standing cases, because of the inability of a childs
skin to handle the traction weight and difficulty maintaining Halter traction at increased weights. The CT
scans show an atlantoaxial rotatory fixation of C1 on C2, which can happen after acute trauma. Given the
long-standing nature of the problem, a soft cervical collar will be of no benefit but may be useful in acute
cases associated with upper respiratory tract infections. Sternocleidomastoid release is used in cases of
congenital muscular torticollis, a condition ruled out by this childs history. Posterior cervical fusion is
reserved for children who have failed nonsurgical treatment such as traction.

2013 American Academy of Orthopaedic Surgeons 2013 Pediatric OrthopaedicSelf-Assessment Examination


2013 Pediatric Orthopaedic Self-Assessment Examination Answer Book 17

RECOMMENDED READINGS
Bowen RE, Mah JY, Otsuka NY. Midshaft clavicle fractures associated with atlantoaxial rotatory
displacement: a report of two cases. J Orthop Trauma. 2003 Jul;17(6):444-7. Review. PubMed PMID:
12843731.
Goddard NJ, Stabler J, Albert JS. Atlanto-axial rotatory fixation and fracture of the clavicle. An
association and a classification. J Bone Joint Surg Br. 1990 Jan;72(1):72-5. Erratum in: J Bone Joint Surg
Br 1990 May;72(3):545. PubMed PMID: 2298798.
Pang D, Li V. Atlantoaxial rotatory fixation: Part 1--biomechanics of normal rotation at the atlantoaxial
joint in children. Neurosurgery. 2004 Sep;55(3):614-25; discussion 625-6. PubMed PMID: 15335428.
Pang D, Li V. Atlantoaxial rotatory fixation: part 2--new diagnostic paradigm and a new classification
based on motion analysis using computed tomographic imaging. Neurosurgery. 2005 Nov;57(5):941-53;
discussion 941-53. PubMed PMID: 16284564.
Pang D, Li V. Atlantoaxial rotatory fixation: part 3-a prospective study of the clinical manifestation,
diagnosis, management, and outcome of children with alantoaxial rotatory fixation. Neurosurgery. 2005
Nov;57(5):954-72; discussion 954-72. PubMed PMID: 16284565.

2013 American Academy of Orthopaedic Surgeons 2013 Pediatric Orthopaedic Self-Assessment Examination
18 American Academy of Orthopaedic Surgeons

Figure 7a Figure 7b

CLINICAL SITUATION FOR QUESTIONS 7 AND 8


Figures 7a and 7b are the MRI and CT scans of a 12-year-old boy who has had 1 week of low-back pain, a
low-grade fever, and an elevated erythrocyte sedimentation rate and white blood cell count.

Question 7
What is the most likely diagnosis?

1. Osteomyelitis of the spine


2. Aneurysmal bone cyst
3. Herniated nucleus pulposus
4. Hemangioma

PREFERRED RESPONSE: 1

Question 8
What is the most appropriate next treatment step?

1. Open biopsy
2. Extension casting
3. CT scan of the chest
4. Blood culture and image-guided needle biopsy or aspiration

PREFERRED RESPONSE: 4

2013 American Academy of Orthopaedic Surgeons 2013 Pediatric OrthopaedicSelf-Assessment Examination


2013 Pediatric Orthopaedic Self-Assessment Examination Answer Book 19

DISCUSSION FOR QUESTIONS 7 AND 8


Based on the patients signs and symptoms, the most likely diagnosis is osteomyelitis of the spine.
The imaging studies do not reveal a herniated disk, which would be unusual in someone of this age.
Aneurysmal bone cysts would involve the posterior elements with a more expansive lesion of bone,
whereas this lesion involves primarily the body and anterior column. Hemangioma would likely not
be present with systemic signs or symptoms and has a more characteristic coarse trabecular pattern
on radiographs and scans. Given the likelihood that this patient has osteomyelitis of the spine, blood
cultures may yield an organism about 50% of the time. If cultures are negative, an image-guided biopsy
is warranted. A diagnosis needs to be established before treatment such as extension casting or bracing
is rendered. Open biopsy may still be needed if needle biopsy is not diagnostic, but this is not the first
treatment option. A chest CT scan is not appropriate as a first step when a tumor is presumed and a
diagnosis has not been established.

RECOMMENDED READINGS FOR QUESTIONS 7 AND 8


Kaplan SL. Osteomyelitis in children. Infect Dis Clin North Am. 2005 Dec;19(4):787-97, vii. Review.
PubMed PMID: 16297732.
Song KM, Sloboda JF. Acute hematogenous osteomyelitis in children. J Am Acad Orthop Surg. 2001
May-Jun;9(3):166-75. PubMed PMID: 11421574.
Dormans JP, Moroz L. Infection and tumors of the spine in children. J Bone Joint Surg Am. 2007 Feb;89
Suppl 1:79-97. Review. PubMed PMID: 17272426.

END OF SERIES

2013 American Academy of Orthopaedic Surgeons 2013 Pediatric Orthopaedic Self-Assessment Examination
20 American Academy of Orthopaedic Surgeons

Figure 9a Figure 9b

Question 9
Figures 9a and 9b are the anteroposterior and lateral tibia/fibula radiographs of a 6-month-old infant who
was evaluated for a bowed leg. In addition to brace treatment, initial treatment should include

1. cast immobilization in a long-leg cast for 4 to 6 weeks to heal the fracture.


2. open reduction and fixation because of the amount of deformity.
3. skeletal survey and referral to social services for evaluation of abuse.
4. referral to genetics to evaluate for a systemic disease.

PREFERRED RESPONSE: 4

DISCUSSION
The radiographs show a severe pseudarthrosis tibia/fibula that has fractured. This is highly associated
with neurofibromatosis type I, which needs to be evaluated with a genetics workup before discussing
treatment. Cast immobilization is not needed if the child is not walking and not in pain. Surgical
management should proceed only after the workup is complete. This is a congenital condition, not a
fracture, so workup for abuse is not indicated.

RECOMMENDED READINGS
Feldman DS, Jordan C, Fonseca L. Orthopaedic manifestations of neurofibromatosis type 1. J Am Acad
Orthop Surg. 2010 Jun;18(6):346-57. Review. PubMed PMID: 20511440.
Vander Have KL, Hensinger RN, Caird M, Johnston C, Farley FA. Congenital pseudarthrosis of the tibia. J
Am Acad Orthop Surg. 2008 Apr;16(4):228-36. Review. PubMed PMID: 18390485.

2013 American Academy of Orthopaedic Surgeons 2013 Pediatric OrthopaedicSelf-Assessment Examination


2013 Pediatric Orthopaedic Self-Assessment Examination Answer Book 21

Figure 10a Figure 10b

Question 10
Figures 10a and 10b are the sagittal and coronal MRI scans of a 5-year-old boy who noticed clicking in
his right knee. His family denied any trauma, but admitted that the child was active and fell frequently.
Birth and developmental history were unremarkable, and specifically negative for other musculoskeletal
conditions. On physical examination, there was no warmth, tenderness, or erythema, or effusion. The
child had an audible and palpable clunk when the knee was taken from a position of extreme flexion to
full extension. There was no anterior, posterior, medial, or lateral instability on examination or medial
or lateral joint line tenderness. The child had not been systemically ill. Radiographs were unrevealing.
What is the most likely diagnosis?

1. Discoid lateral meniscus


2. Congenital absence of the anterior cruciate ligament
3. Torn medial meniscus
4. Osteomyelitis of the distal femur

PREFERRED RESPONSE: 1

DISCUSSION
A discoid lateral meniscus is probably the most common cause of a symptomatic clicking or clunking
in the knee in a young child. This is a congenital problem that can become symptomatic as soon as a
child ambulates, or the condition may remain asymptomatic for several years. The meniscus develops
from a cartilaginous anlage and at no point in its development is it discoid. The MRI scans reveal a band
of meniscal tissue filling the joints lateral compartment on both sagittal and coronal images. A medial
meniscal tear is usually accompanied by a history of injury and an effusion, which are not present in this
child. There is also no joint line tenderness, which makes this diagnosis less likely. Congenital absence of
the anterior cruciate ligament may be found in children born with congenital knee hyperextension, which
is ruled out in this case by normal history and examination findings. Children with osteomyelitis are often
systemically ill. On examination, they may have warmth and tenderness. MRI scans will often show an
area of increased signal on T1-weighted images.

2013 American Academy of Orthopaedic Surgeons 2013 Pediatric Orthopaedic Self-Assessment Examination
22 American Academy of Orthopaedic Surgeons

RECOMMENDED READINGS
Kramer DE, Micheli LJ. Meniscal tears and discoid meniscus in children: diagnosis and treatment. J Am
Acad Orthop Surg. 2009 Nov;17(11):698-707. Review. PubMed PMID: 19880680.
Stanitski CL. Discoid meniscus. In: Micheli LJ, Kocher MS, eds. The Pediatric and Adolescent Knee.
Philadelphia, PA: Elsevier; 2006:260-272.
Watanabe M, Takeda S, Ikeuchi H. Atlas of Arthroscopy. 3rd ed. Tokyo, Japan: Springer-Verlag; 1979:75-
130.

Figure 11

Question 11
Figure 11 is the radiograph of a 9-year-old African-American boy with left-sided groin and knee pain. His
body mass index is 42, and he has had symptoms for 10 days. What is the recommended treatment?

1. Physical therapy and a return visit in 6 weeks


2. Crutches and toe-touch weight bearing for 6 weeks
3. Open hip dislocation and a modified Dunn procedure
4. Percutaneous screw fixation of both hips

PREFERRED RESPONSE: 4

DISCUSSION
This patient is young, obese, and African American. His risk of slip on the right is at least 50%. The
radiograph shows a grade 1 slip on the left and an open triradiate cartilage. Recommendations for this
patient should include fixation of both hips because risk for problems after a slip of the opposite side
is higher than risk for prophylactic fixation. Physical therapy may make the condition worse. Toe-
touch weight bearing would only delay needed treatment. The left hip should be treated and the family
may elect to only treat the left hip, but recommendations should be given to treat both hips. Open-hip
dislocation is not indicated in this mild slipped capital femoral epiphysis.

2013 American Academy of Orthopaedic Surgeons 2013 Pediatric OrthopaedicSelf-Assessment Examination


2013 Pediatric Orthopaedic Self-Assessment Examination Answer Book 23

RECOMMENDED READINGS
Loder RT. Correlation of radiographic changes with disease severity and demographic variables in
children with stable slipped capital femoral epiphysis. J Pediatr Orthop. 2008 Apr-May;28(3):284-90.
PubMed PMID: 18362791.
Yildirim Y, Bautista S, Davidson RS. Chondrolysis, osteonecrosis, and slip severity in patients with
subsequent contralateral slipped capital femoral epiphysis. J Bone Joint Surg Am. 2008 Mar;90(3):485-92.
PubMed PMID: 18310697.

Figure 12

Question 12
Figure 12 is the radiograph of a 3-year-old who has deformity of the first ray of the right foot. What is the
most appropriate recommendation at this time?

1. Observation
2. Excision of the abnormal structure
3. Lengthening of the metatarsal with external fixation
4. Ray resection

PREFERRED RESPONSE: 2

DISCUSSION
The oblique radiograph shows a well-formed congenital longitudinal epiphyseal bracket. Excision of the
bracket leads to decreased deformity and improves the potential for longitudinal growth. Observation
alone will lead to the relative shortening of the first metatarsal and medial deviation of the hallux.
Metatarsal lengthening can be considered if resection does not lead to sufficient growth. Ray resection is
not necessary.

2013 American Academy of Orthopaedic Surgeons 2013 Pediatric Orthopaedic Self-Assessment Examination
24 American Academy of Orthopaedic Surgeons

RECOMMENDED READINGS
Mubarak SJ, OBrien TJ, Davids JR. Metatarsal epiphyseal bracket: treatment by central physiolysis. J
Pediatr Orthop. 1993 Jan-Feb;13(1):5-8. PubMed PMID: 8416354.
Shea KG, Mubarak SJ, Alamin T. Preossified longitudinal epiphyseal bracket of the foot: treatment by
partial bracket excision before ossification. J Pediatr Orthop. 2001 May-Jun;21(3):360-5. PubMed PMID:
11371821.

Figure 13

Question 13
Figure 13 is the photograph of 18-month-old triplets with a lower-limb condition. What is the best initial
treatment?

1. Vitamin D supplementation and serum level monitoring


2. Mechanical axis alignment measurements on serial standing knee radiographs
3. Genetic testing and counseling for short-stature syndrome
4. Observation and clinical follow-up.

PREFERRED RESPONSE: 4

DISCUSSION
These triplets exhibit genu varum and internal tibial torsion that can be part of normal development. Fetal
packing is the likely major contributing cause for these triplets, however. Observation and follow-up will
be sufficient. Bowing and torsion can be clinical features of vitamin D deficiency, Blount disease, and
short-stature syndromes, but these are not the most likely diagnoses. Radiographic diagnosis of Blount
disease may not be accurate at this age.

RECOMMENDED READINGS
Kling TF Jr, Hensinger RN. Angular and torsional deformities of the lower limbs in children. Clin Orthop
Relat Res. 1983 Jun;(176):136-47. PubMed PMID: 6851317.
Staheli LT, Engel GM. Tibial torsion: a method of assessment and a survey of normal children. Clin
Orthop Relat Res. 1972 Jul-Aug;86:183-6. PubMed PMID: 5047787.
Staheli LT, Corbett M, Wyss C, King H. Lower-extremity rotational problems in children. Normal values
to guide management. J Bone Joint Surg Am. 1985 Jan;67(1):39-47. PubMed PMID: 3968103.

2013 American Academy of Orthopaedic Surgeons 2013 Pediatric OrthopaedicSelf-Assessment Examination


2013 Pediatric Orthopaedic Self-Assessment Examination Answer Book 25

Figure 14a Figure 14b

Figure 14c Figure 14d

Question 14
Figures 14a through 14d are the radiographs and sagittal T1- and T2-weighted MRI scans of an otherwise
healthy 10-year-old girl who has experienced 3 weeks of worsening pain with weight bearing and is now
refusing to bear weight to either of her lower extremities. She denies any history of injury or trauma.
She has not had any fevers or recent illnesses. She denies any numbness or parasthesias. She has had
no bowel or bladder incontinence. In the emergency department she is afebrile and tender to palpation
over the midlumbar spine. She has pain with hyperextension and flexion of her back. Her white blood
cell count is 11.4 (reference range, 4.5-11.0 x109/L), erythrocyte sedimentation rate is 40 mm/h (reference
range, 0-20 mm/h), and C-reactive protein is 2.6 mg/L (reference range, 0.08-3.1 mg/L). What is the most
appropriate course of action at this time?

1. Admission and traction


2. Open surgical debridement of the lesion
3. Perform an open biopsy of the lesion
4. Nonsurgical treatment with antibiotics and immobilization

PREFERRED RESPONSE: 4

DISCUSSION
Childhood diskitis represents one end of a continuum of spinal infections, from diskitis to vertebral
osteomyelitis with soft-tissue abscess. Diskitis is now generally accepted as a bacterial infection
involving the disk space and adjacent vertebral end plates. Optimal treatment includes the use of
antibiotics. Empiric coverage is directed against Staphylococcus aureus because it has been the most
common organism isolated from culture-positive biopsy specimens. Open biopsy is not necessary for
patients who exhibit all of these clinical characteristics, but it is indicated for those whose symptoms do
not resolve rapidly with treatment or who have atypical presentation.
2013 American Academy of Orthopaedic Surgeons 2013 Pediatric Orthopaedic Self-Assessment Examination
26 American Academy of Orthopaedic Surgeons

RECOMMENDED READINGS
Ring D, Johnston CE 2nd, Wenger DR. Pyogenic infectious spondylitis in children: the convergence
of discitis and vertebral osteomyelitis. J Pediatr Orthop. 1995 Sep-Oct;15(5):652-60. PubMed PMID:
7593581.
Hensinger RN. Acute back pain in children. Instr Course Lect. 1995;44:111-26. Review. PubMed PMID:
7797849.
Early SD, Kay RM, Tolo VT. Childhood diskitis. J Am Acad Orthop Surg. 2003 Nov-Dec;11(6):413-20.
Review. PubMed PMID: 14686826.

Figure 15

Question 15
Figure 15 is the photograph of a 3-year-old girl. She has intoeing (30-degree internal foot progression
angle) and tripping when running fast. Prone examination reveals a neutral thigh foot angle and hips that
rotate internally 85 degrees and externally 15 degrees. She has no pain; the remainder of her examination
and history is unremarkable. What is the most appropriate treatment?

1. Surgical derotation osteotomy of the femurs


2. Shoe inserts
3. A nighttime external rotation brace and physical therapy
4. Observation

PREFERRED RESPONSE: 4

DISCUSSION
This patient has femoral anteversion, which peaks at around this age. An awkward intoeing gait and W
sitting are common complaints, but pain is rare. Observation is recommended. Special shoes, therapy,
and derotating braces have never been shown to improve rates of remodeling, which approach 99% by age
8 to 10.

RECOMMENDED READINGS
Aronsson DD. The pediatric orthopaedic examination. In: Morrissy RT, Weinstein SL, eds. Lovell and
Winters Pediatric Orthopaedics. 6th ed. Philadelphia, PA: Lippincott William & Wilkins; 2006:1213-
1256.
Blackmur JP, Murray AW. Do children who in-toe need to be referred to an orthopaedic clinic? J Pediatr
Orthop B. 2010 Sep;19(5):415-7. PubMed PMID: 20520580.

2013 American Academy of Orthopaedic Surgeons 2013 Pediatric OrthopaedicSelf-Assessment Examination


2013 Pediatric Orthopaedic Self-Assessment Examination Answer Book 27

Question 16
A 1-week-old infant was placed in a Pavlik harness for an Ortolani-positive hip. She was seen on a
weekly basis and her hip remained dislocated 3 weeks later. What is the most appropriate treatment?

1. Continue the Pavlik harness


2. Switch to an abduction orthosis
3. Plan for closed reduction of the hip
4. Schedule for open reduction

PREFERRED RESPONSE: 2

DISCUSSION
Two series have documented successful hip reduction with rigid abduction orthoses after the failure
of Pavlik harness treatment. If a dislocated hip has not stabilized with 3 weeks of Pavlik treatment,
prolonging treatment can lead to Pavlik disease, or erosion of the posterior margin of the acetabulum.
Closed or open reduction should be deferred in favor of a trial of abduction bracing.

RECOMMENDED READINGS
Hedequist D, Kasser J, Emans J. Use of an abduction brace for developmental dysplasia of the hip after
failure of Pavlik harness use. J Pediatr Orthop. 2003 Mar-Apr;23(2):175-7. PubMed PMID: 12604946.
Swaroop VT, Mubarak SJ. Difficult-to-treat Ortolani-positive hip: improved success with new treatment
protocol. J Pediatr Orthop. 2009 Apr-May;29(3):224-30. PubMed PMID: 19305270.

2013 American Academy of Orthopaedic Surgeons 2013 Pediatric Orthopaedic Self-Assessment Examination
28 American Academy of Orthopaedic Surgeons

Figure 17a Figure 17b Figure 17c

Figure 17d Figure 17e Figure 17f

Question 17
A 15-year-old boy fell from a tree and sustained the injury shown in Figures 17a and 17b. He was treated
with closed manipulation and splinting. His radiographs after manipulation are shown in Figures 17c and
17d. His follow-up radiographs, taken 1 week later, are shown in Figures 17e and 17f. At this current
follow-up visit, which treatment recommendation is most appropriate?

1. Acceptance of the current deformity, with the expectation that his bone will remodel
2. Remanipulation and splinting of the fractures using a hematoma block in the office
3. Remanipuation and casting in the operating room under general anesthesia
4. Open reduction and internal fixation of the radius fracture under general anesthesia

PREFERRED RESPONSE: 4

DISCUSSION
The radiographs show a distal radial shaft fracture. The distal radial physis appears to be nearly or
completely closed, so significant remodeling of the residual bayonette apposition should not be expected.
The patient has already attempted manipulation, and remanipulation, whether in an office setting or under
general anesthesia, is unlikely to maintain an adequate reduction. In this age group treatment of radial
shaft fractures with internal fixation provides the most definitive outcome, posing the least risk for repeat
surgery, and allows early rehabilitation.

2013 American Academy of Orthopaedic Surgeons 2013 Pediatric OrthopaedicSelf-Assessment Examination


2013 Pediatric Orthopaedic Self-Assessment Examination Answer Book 29

RECOMMENDED READINGS
Hertel R, Pisan M, Lambert S, Ballmer FT. Plate osteosynthesis of diaphyseal fractures of the radius and
ulna. Injury. 1996 Oct;27(8):545-8. PubMed PMID: 8994558.
Chow SP, Leung F. Radial and ulnar shaft fractures. In: Bucholz RW, Heckman JD, Court-Brown CM,
Tornetta P, eds. Rockwood and Greens Fractures in Adults. 7th ed. Philadelphia, PA: Lippincott Williams
& Wilkins; 2010:chap 31.

Question 18
A displaced pediatric supracondylar humerus fracture is treated with closed manipulation and placement
of 2 Kirschner wires placed from the lateral side. What would be the effect of adding a third pin from the
lateral side?

1. Increase risk for iatrogenic ulnar nerve injury


2. Provide more construct stiffness than adding a medial pin
3. Improve construct stiffness in the presence of medial column comminution
4. Will not affect construct stiffness in the presence of residual distal fragment internal rotation

PREFERRED RESPONSE: 2

DISCUSSION
Multiple biomechanical studies have shown that the addition of a third pin from the lateral side improves
construct stiffness in the presence of medial column loss or slight internal rotation of the distal fragment.
The same studies show that addition of a medial pin (cross pinning) has essentially the same benefit.
Placement of a medial pin increases risk for iatrogenic nerve injury.

RECOMMENDED READINGS
Larson L, Firoozbakhsh K, Passarelli R, Bosch P. Biomechanical analysis of pinning techniques for
pediatric supracondylar humerus fractures. J Pediatr Orthop. 2006 Sep-Oct;26(5):573-8. PubMed PMID:
16932093.
Bloom T, Robertson C, Mahar AT, Newton P. Biomechanical analysis of supracondylar humerus fracture
pinning for slightly malreduced fractures. J Pediatr Orthop. 2008 Oct-Nov;28(7):766-72. PubMed PMID:
18812905.
Babal JC, Mehlman CT, Klein G. Nerve injuries associated with pediatric supracondylar humeral
fractures: a meta-analysis. J Pediatr Orthop. 2010 Apr-May;30(3):253-63. PubMed PMID: 20357592.

2013 American Academy of Orthopaedic Surgeons 2013 Pediatric Orthopaedic Self-Assessment Examination
30 American Academy of Orthopaedic Surgeons

Question 19
Nonossifying fibroma (NOF) is best characterized as

1. an eccentric scalloped metaphyseal radiolucent lesion.


2. producing a fallen leaf sign.
3. a central metaphyseal radiolucent lesion with fusiform expansion to the width of the physis.
4. having a ground glass appearance.

PREFERRED RESPONSE: 1

DISCUSSION
NOF lesions are eccentric and scalloped. Unicameral bone cysts (UBC) are central cystic lesions that
expand to the width of the physis. When UBCs fracture, a fragment of bone can be seen in the cyst; this is
called the fallen leaf sign. A ground glass appearance is seen in fibrous dysplasia.

RECOMMENDED READINGS
Wyers MR. Evaluation of pediatric bone lesions. Pediatr Radiol. 2010 Apr;40(4):468-73. Review. PubMed
PMID: 20225104.
Helms CA, ed. Fundamentals of Skeletal Radiology. Philadelphia, PA: WB Saunders; 1995:18-21.

RESPONSES FOR QUESTIONS 20 THROUGH 27


1. Preoperative evaluation by cardiology and pulmonary
2. Preoperative evaluation by pulmonary and nutrition
3. MRI scan of spinal axis
4. Preoperative evaluation by a hematologist
5. Flexion/extension lateral C-spine radiographs
6. Preoperative evaluation by cardiology and an MRI scan of the spinal axis

For each of the clinical scenarios below, match the most appropriate preoperative work-up or
evaluation.

Question 20
A 14-year-old girl with a 65-degree scoliosis with caf au lait cutaneous lesions and axillary freckling.

PREFERRED RESPONSE: 3

2013 American Academy of Orthopaedic Surgeons 2013 Pediatric OrthopaedicSelf-Assessment Examination


2013 Pediatric Orthopaedic Self-Assessment Examination Answer Book 31

Question 21
A 14-year-old boy with Duchene muscular dystrophy and a 48-degree scoliosis.

PREFERRED RESPONSE: 1

Question 22
A 13-year-old girl 1 year postmenarche with a 58-degree left thoracic curve.

PREFERRED RESPONSE: 3

Question 23
A 15-year-old tall, thin girl with positive wrist and thumb signs and a 65-degree scoliosis.

PREFERRED RESPONSE: 6

Question 24
A 16-year-old with cerebral palsy and an 80-degree neuromuscular curve with a body weight below the
5th percentile.

PREFERRED RESPONSE: 2

Question 25
A 9-year-old girl with a 57-degree early-onset scoliosis who failed bracing preoperatively for growing
rods.

PREFERRED RESPONSE: 3

Question 26
An 8-year-old girl with Morquio syndrome with a 65-degree scoliosis for growing rods.

PREFERRED RESPONSE: 5

2013 American Academy of Orthopaedic Surgeons 2013 Pediatric Orthopaedic Self-Assessment Examination
32 American Academy of Orthopaedic Surgeons

Question 27
A 13-year-old boy with a 58-degree scoliosis and Down syndrome.

PREFERRED RESPONSE: 5

DISCUSSION FOR QUESTIONS 20 THROUGH 27


Scoliosis in the setting of these cutaneous abnormalities is likely occurring in a patient with
neurofibromatosis. Prior to surgery in patients with neurofibromatosis, a preoperative MRI scan is
helpful to assess for dural ectasia and intraspinal neurofibromas. Patients with Duchenne muscular
dystrophy develop impairment of both pulmonary and cardiac function and need pulmonary and cardiac
evaluation on an ongoing basis. Spinal fusion surgery should not be performed in these patients without
cardiology and pulmonary evaluations before surgery. Left thoracic curves, like all atypical scoliosis
conditions, require evaluation before surgery with an MRI scan of the spinal axis to rule-out intraspinal
canal abnormalities such as a syrinx. Scoliosis in the setting of arachnodactyly strongly suggests Marfan
syndrome. Patients with Marfan syndrome may have dural ectasia and require an MRI scan before spinal
fusion surgery. In addition, cardiac abnormalities may also be present, and echocardiography is indicated
to rule-out aortic abnormalities. A child with cerebral palsy should be evaluated by both pulmonary
and the nutrition service before spinal fusion surgery, especially if the patients weight is below the 5th
percentile. Although formal pulmonary function testing may not be possible in patients with cerebral
palsy, evaluation of the patients cough and general pulmonary status is important to assess surgical risk
and postsurgical pulmonary problems. Early-onset scoliosis constitutes atypical scoliosis, and studies
have confirmed a higher incidence of intraspinal canal abnormalities in patients with atypical scoliosis.
An MRI scan is indicated to rule out intraspinal pathology before instrumenting the spine in these
patients. Skeletal dysplasia syndromes may be associated with ligamentous laxity and C1-C2 instability,
and this is specifically associated with Morquio mucopolysaccharidosis syndrome. Upper-cervical-spine
instability needs to be investigated before proceeding with spinal fusion for scoliosis. Radial longitudinal
deficiencies are associated with thrombocytopenia and Fanconi anemia, and careful evaluation before any
surgeries are performed in these patients is needed to preclude a bleeding disorder. Down syndrome is
associated with ligamentous laxity and upper-cervical-spine instability. Ruling-out C1-C2 instability is
necessary for these patients before spinal fusion.

2013 American Academy of Orthopaedic Surgeons 2013 Pediatric OrthopaedicSelf-Assessment Examination


2013 Pediatric Orthopaedic Self-Assessment Examination Answer Book 33

RECOMMENDED READINGS FOR QUESTIONS 20 THROUGH 27


Luhmann SJ, Skaggs DL. Pediatric spine conditions. In: Lieberman JR, ed. AAOS Comprehensive
Orthopaedic Review. Rosemont, IL: American Academy of Orthopaedic Surgeons; 2009:245-265.
Zane MSM. Pediatric neuromuscular disorders. In: Lieberman JR, ed. AAOS Comprehensive Orthopaedic
Review. Rosemont, IL: American Academy of Orthopaedic Surgeons; 2009:293-309.
Bae DS. Congenital hand and wrist anomalies and brachial plexus palsies. In: Lieberman JR, ed. AAOS
Comprehensive Orthopaedic Review. Rosemont, IL: American Academy of Orthopaedic Surgeons;
2009:311-320.
Spencer SA. Connective tissue diseases, arthritides, and other diseases. In: Lieberman JR, ed. AAOS
Comprehensive Orthopaedic Review. Rosemont, IL: American Academy of Orthopaedic Surgeons;
2009:367-377.
McCarthy JJ, DAndrea LP, Betz RR, Clements DH. Scoliosis in the child with cerebral palsy. J Am Acad
Orthop Surg. 2006 Jun;14(6):367-75. Review. PubMed PMID: 16757676.
Gillingham BL, Fan RA, Akbarnia BA. Early onset idiopathic scoliosis. J Am Acad Orthop Surg. 2006
Feb;14(2):101-12. Review. PubMed PMID: 16467185.

END OF SERIES

Figure 28

Question 28
What is the most likely complication associated with the injury shown in Figure 28?

1. Infection
2. Malunion with development of varus deformity
3. Chondrolysis
4. Osteonecrosis of the femoral head

PREFERRED RESPONSE: 4

2013 American Academy of Orthopaedic Surgeons 2013 Pediatric Orthopaedic Self-Assessment Examination
34 American Academy of Orthopaedic Surgeons

DISCUSSION
This radiograph shows a Delbet type II fracture of the femoral neck. Osteonecrosis is the most
common complication following hip fracture in children. Varus deformity is the second-most-common
complication. Nonunion occurs less frequently than coxa vara. Chondrolysis and infection are relatively
rare complications. Osteonecrosis is most likely the result of vascular injury that occurs at the time
of fracture. Type I, II, and III fractures were 15, 6, and 4 times more likely, respectively, to result in
osteonecrosis than were type IV fractures. Disruption of blood flow may be caused by kinking or tearing
of the vessels occurring with fracture displacement or by direct laceration of blood vessels by fracture
fragments.

RECOMMENDED READINGS
Boardman MJ, Herman MJ, Buck B, Pizzutillo PD. Hip fractures in children. J Am Acad Orthop Surg.
2009 Mar;17(3):162-73. Review. PubMed PMID: 19264709.
Beaty JH. Fractures of the hip in children. Orthop Clin North Am. 2006 Apr;37(2):223-32, vii. Review.
PubMed PMID: 16638453.

Figure 29

Question 29
Figure 29 is the photograph of a 7-month-old infant. What is the mode of inheritance?

1. X-linked recessive
2. Autosomal-recessive
3. Maternal mitochondrial DNA
4. Autosomal-dominant

PREFERRED RESPONSE: 4

2013 American Academy of Orthopaedic Surgeons 2013 Pediatric OrthopaedicSelf-Assessment Examination


2013 Pediatric Orthopaedic Self-Assessment Examination Answer Book 35

DISCUSSION
Postaxial polydactyly is a common autosomal-dominant trait seen in approximately 1 in 200 births in the
United States, usually with a positive family history, although foot polydactyly is less common than hand
polydactyly. The other modes of inheritance are not applicable. This condition is not associated with
other systemic disorders when the remainder of the physical examination is normal, so no further workup
is needed. Children with postaxial polydactyly of the foot exhibit normal motor milestones. Elective
surgical reconstruction is performed to improve cosmesis and shoe wearing, and can be done at any age in
accordance with the cultural and social expectations of the family.

RECOMMENDED READINGS
Kasser JR. The foot. In: Morrissy RT, Weinstein SL, eds. Lovell and Winters Pediatric Orthopaedics. 6th
ed. Philadelphia, PA: Lippincott Williams & Wilkins; 2006:1257-1328.
Phelps DA, Grogan DP. Polydactyly of the foot. J Pediatr Orthop. 1985 Jul-Aug;5(4):446-51. PubMed
PMID: 4019759.

Figure 30a Figure 30b

Question 30
Figures 30a and 30b are the radiographs of a 10-year-old ambulatory patient with arthrogryposis
who fell 2 days ago and sustained an injury to the left knee. Physical examination revealed pain
in the anterior knee, a large knee joint effusion, and an inability to straighten the knee. What is the
most appropriate treatment?

1. Open reduction and internal fixation of the fragments with a suture placed in the
quadriceps tendon and through patellar bone tunnels
2. Placement of an above-knee cast with the knee extended, and limited weight bearing
for 6 weeks
3. Placement of a knee immobilizer and weight bearing as tolerated for 6 weeks
4. Open reduction and direct repair of the quadriceps tendon with sutures

PREFERRED RESPONSE: 1

2013 American Academy of Orthopaedic Surgeons 2013 Pediatric Orthopaedic Self-Assessment Examination
36 American Academy of Orthopaedic Surgeons

DISCUSSION
The radiographs reveal a small rim of bone avulsed proximally from the patella. This represents a
superior patellar sleeve fracture. In an ambulatory patient the extensor mechanism must be repaired to
function properly. Consequently, immobilization only, either in a cast or knee immobilizer, would not
be appropriate. The fracture is distal to the quadriceps tendon insertion. The proximal avulsed fragment
likely contains a significant portion of cartilage, which cannot be appreciated on the radiographs. The
repair should be made between the bony fragments, usually through bone tunnels in the larger distal
fragment.

RECOMMENDED READINGS
Gettys FK, Morgan RJ, Fleischli JE. Superior pole sleeve fracture of the patella: a case report and review
of the literature. Am J Sports Med. 2010 Nov;38(11):2331-6. Epub 2010 Sep 1. Review. PubMed PMID:
20810782.
Maripuri SN, Mehta H, Mohanty K. Sleeve fracture of the superior pole of the patella with an intra-
articular dislocation. A case report. J Bone Joint Surg Am. 2008 Feb;90(2):385-9. PubMed PMID:
18245599.

2013 American Academy of Orthopaedic Surgeons 2013 Pediatric OrthopaedicSelf-Assessment Examination


2013 Pediatric Orthopaedic Self-Assessment Examination Answer Book 37

Figure 31

Question 31
Figure 31 is the T2-weighted MRI scan of an 11-year-old boy with a 7-day history of limping, pain in his
right leg, and fevers. His MRI scan reveals soft-tissue swelling. The distal femur is tender to palpation,
but there is no pain with hip or knee range of motion. He is unable to bear weight on his right leg. His
temperature is 38.8C, pulse 110, and blood pressure 120/70 mm Hg. Lab study results revealed a white
blood cell count of 16000 cells/L (reference range, 4500-11000 cells/L), hematocrit 30% (reference
range, 41%-50%), and C-reactive protein of 20 mg/L (reference range, 0.08-3.1 mg/L). A gram stain from
a blood culture obtained in the emergency department showed gram plus cocci in clusters. What is the
most appropriate empiric antibiotic treatment?

1. Vancomycin
2. Cephalexin
3. Oxacillin
4. Imipenem
PREFERRED RESPONSE: 1

DISCUSSION
This patient has findings consistent with acute osteomyelitis caused by Staphylococcus aureus. His
clinical presentation strongly suggests the causative organism will be methicillin-resistant Staphylococcus
aureus (MRSA). A recent study identified 4 independent predictors (temperature > 38.0 C, white blood-
cell count > 12000 cells/L, hematocrit < 34%, and C-reactive protein > 13 mg/L) to reliably differentiate
between MRSA and MSSA osteomyelitis. The predicted probability of MRSA osteomyelitis is 92% if
all 4 predictors are present (as in this case), 45% if 3 are present, 10% if 2 are present, 1% if 1 predictor
is present. Empiric treatment should include an antibiotic active against MRSA, such as vancomycin.
Oxacillin, cephalexin, and imipenem are active against MSSA, but not MRSA.

2013 American Academy of Orthopaedic Surgeons 2013 Pediatric Orthopaedic Self-Assessment Examination
38 American Academy of Orthopaedic Surgeons

RECOMMENDED READINGS
Ju KL, Zurakowski D, Kocher MS. Differentiating between methicillin-resistant and methicillin-sensitive
Staphylococcus aureus osteomyelitis in children: an evidence-based clinical prediction algorithm. J Bone
Joint Surg Am. 2011 Sep 21;93(18):1693-701. PubMed PMID: 21938373.
Copley LA. Pediatric musculoskeletal infection: trends and antibiotic recommendations. J Am Acad
Orthop Surg. 2009 Oct;17(10):618-26. Review. PubMed PMID: 19794219.

Figure 32a Figure 32b Figure 32c

CLINICAL SITUATION FOR QUESTIONS 32 AND 33


Figures 32a through 32c are the radiograph, bone scan, and MRI scan of a 15-month-old infant.
Intravenous antibiotics were initiated 2 days ago in response to blood cultures that were positive for
Staphylococcus aureus. Since antibiotic initiation, his fever dropped from a maximum of 39.4C to
37.2C with occasional mild spikes. The hospitalist noticed that the babys left upper extremity was not
moving.

Question 32
What is the next step in orthopaedic management of this condition?

1. Vital sign monitoring every 2 hours for temperature spikes


2. Peripheral indwelling central catheter placement for 6 weeks of intravenous antibiotics
3. CT scan to further delineate metaphyseal changes
4. Drainage of the gleno-humeral joint and debridement of the humerus

PREFERRED RESPONSE: 4

2013 American Academy of Orthopaedic Surgeons 2013 Pediatric OrthopaedicSelf-Assessment Examination


2013 Pediatric Orthopaedic Self-Assessment Examination Answer Book 39

Question 33
What do the bone scan findings represent?

1. Sequestrum
2. Involucrum
3. Osteonecrosis
4. Heterotopic ossification

PREFERRED RESPONSE: 3

DISCUSSION FOR QUESTIONS 32 AND 33


The bone scan shows no uptake of the tracer, which indicates osteonecrosis. The MRI scan of the
shoulder reveals sepsis with a focus on osteomyelitis. A CT scan will not add more useful information and
will delay treatment. A sequestrum is a piece of dead bone that has become separated from normal/solid
bone during the osteonecrosis process. It appears as a radiopacity on plain radiograph. The involucrum
is new bone formed by an elevated periosteum and can be seen on radiograph. Heterotopic ossification
appears as radiopacity within the soft tissues. Late recognition of pediatric shoulder sepsis has been
reported, and damage to the joint can be extensive. Septic joints are one of the few true orthopaedic
emergencies. The methodology of drainage in shoulder sepsis is controversial (aspiration vs open
drainage vs arthroscopic lavage), but the joint must be drained and sterilized to prevent cartilage damage.

RECOMMENDED READINGS FOR QUESTIONS 32 AND 33


Lossos IS, Yossepowitch O, Kandel L, Yardeni D, Arber N. Septic arthritis of the glenohumeral joint. A
report of 11 cases and review of the literature. Medicine (Baltimore). 1998 May;77(3):177-87. Review.
PubMed PMID: 9653429.
Forward DP, Hunter JB. Arthroscopic washout of the shoulder for septic arthritis in infants. A new
technique. J Bone Joint Surg Br. 2002 Nov;84(8):1173-5. PubMed PMID: 12463665.
Smith SP, Thyoka M, Lavy CB, Pitani A. Septic arthritis of the shoulder in children in Malawi. A
randomised, prospective study of aspiration versus arthrotomy and washout. J Bone Joint Surg Br. 2002
Nov;84(8):1167-72. PubMed PMID: 12463664.

END OF SERIES

2013 American Academy of Orthopaedic Surgeons 2013 Pediatric Orthopaedic Self-Assessment Examination
40 American Academy of Orthopaedic Surgeons

Question 34
Physical examination of a 6-week-old infant revealed a positive left Galeazzi sign. With hip abduction
to 80 degrees, there was a palpable clunk on the left but none on the right. Ultrasound imaging showed a
right hip alpha angle of 60 degrees with 50% coverage of the femoral head; the left hip alpha angle was 40
degrees with 10% coverage of the femoral head. What is the most appropriate treatment for this problem?

1. Full-time placement of a Pavlik harness with follow-up in 1 week


2. Full-time placement of a Pavlik harness with follow-up examination and ultrasound in 3
months
3. Observation with follow-up at age 6 months with hip radiographs
4. Double diapering with follow-up examination and ultrasound in 1 month

PREFERRED RESPONSE: 1

DISCUSSION
This infant has a developmentally dislocated hip that is reducible (Ortolani positive) with an abnormal
ultrasound. At age 6 weeks, this should be treated with a full-time Pavlik harness. Observation is likely to
lead to a rigidly dislocated hip not amenable to bracing. Double diapering has no treatment effect in true
developmental hip dysplasia/dislocation. Infants with unstable hips need to be checked every 1 to 2 weeks
until the hip is stable. Follow-up in 3 months is not appropriate for an unstable hip.

RECOMMENDED READINGS
Weinstein SL. Developmental hip dysplasia and dislocation. In: Morrissy RT, Weinstein SL, eds. Lovell
and Winters Pediatric Orthopaedics. 6th ed. Philadelphia, PA: Lippincott Williams & Wilkins; 2006:987-
1038.
Murnaghan ML, Browne RH, Sucato DJ, Birch J. Femoral nerve palsy in Pavlik harness treatment for
developmental dysplasia of the hip. J Bone Joint Surg Am. 2011 Mar 2;93(5):493-9. PubMed PMID:
21368082.

Question 35
In patients treated with growing rods for early-onset scoliosis, which of the following outcomes has been
shown to occur over time with subsequent lengthening procedures?

1. Distraction force required increases, length obtained at each procedure decreases


2. Distraction force required increases, length obtained at each procedure increases
3. Distraction force required decreases, length obtained at each procedure increases
4. Distraction force required decreases, length obtained at each procedure decreases

PREFERRED RESPONSE: 1

2013 American Academy of Orthopaedic Surgeons 2013 Pediatric OrthopaedicSelf-Assessment Examination


2013 Pediatric Orthopaedic Self-Assessment Examination Answer Book 41

DISCUSSION
In a prospective study of 60 growing rod lengthening procedures to treat early-onset scoliosis, detailed
measurements of the amount of distraction force required and length gained during the procedure were
taken. The forces required to distract the spine increased over time, and mean length achieved with each
distraction decreased. By the fifth lengthening, required distraction forced had doubled vs baseline, and
fewer than 8 mm of growth was achieved with each subsequent lengthening.

RECOMMENDED READINGS
Noordeen HM, Shah SA, Elsebaie HB, Garrido E, Farooq N, Al-Mukhtar M. In vivo distraction force
and length measurements of growing rods: which factors influence the ability to lengthen? Spine (Phila
Pa 1976). 2011 Dec 15;36(26):2299-303. Erratum in: Spine (Phila Pa 1976). 2012 Mar 1;37(5):432. Al
Mukhtar, Mohannad [corrected to Al-Mukhtar, Mohannad]. PubMed PMID: 21494191.
Farooq N, Garrido E, Altaf F, Dartnell J, Shah SA, Tucker SK, Noordeen H. Minimizing complications
with single submuscular growing rods: a review of technique and results on 88 patients with minimum
two-year follow-up. Spine (Phila Pa 1976). 2010 Dec 1;35(25):2252-8. PubMed PMID: 21102301.

Figure 36

Question 36
Figure 36 is the sitting radiograph of a 12-year-old boy with Duchenne muscular dystrophy who has a
38-degree scoliosis and pulmonary function tests showing a forced vital capacity of 50% of the predicted
percentage. His cardiac status is stable. When is the best time to perform spinal fusion surgery?

1. When his pulmonary function tests decline to 35% of predicted value


2. When his cardiac status starts to decline
3. When his scoliosis reaches 50 degrees
4. As soon as possible

PREFERRED RESPONSE: 4

2013 American Academy of Orthopaedic Surgeons 2013 Pediatric Orthopaedic Self-Assessment Examination
42 American Academy of Orthopaedic Surgeons

DISCUSSION
Timing of surgery in patients with Duchenne muscular dystrophy is based on the patients pulmonary
and cardiac status and not the severity of scoliosis. Because more than 90% of these boys will develop
scoliosis and pulmonary function declines throughout their lives, operating before the pulmonary
function tests decline too significantly enhances the likelihood of successful weaning from the ventilator
postsurgically and minimizes the possibility of a tracheostomy. The Cobb angle measurement is
superseded in this diagnosis by the cardiopulmonary status of the patient, making the response When his
scoliosis reaches 50 degrees wrong. Pulmonary function testing at 35% of predicted value is the absolute
threshold for surgery and a reasonable expectation for a successful wean off the ventilator, so surgery
should ideally be done before this time.

RECOMMENDED READINGS
Zane MSM. Pediatric neuromuscular disorders. In: Lieberman JR, ed. AAOS Comprehensive Orthopaedic
Review. Rosemont, IL: American Academy of Orthopaedic Surgeons; 2009:293-309.
Sussman M. Duchenne muscular dystrophy. J Am Acad Orthop Surg. 2002 Mar-Apr;10(2):138-51.
Review. PubMed PMID: 11929208.

Figure 37

Question 37
Figure 37 is the photograph of a 10-year-old girl standing without shoes who underwent surgical
correction of a left clubfoot deformity at age one. The family is concerned that her left lower extremity is
smaller than the right. What is the etiology of the size difference between the left and right?

1. Tethered cord
2. Lumbar scoliosis
3. Intrinsic hypoplasia of the left lower extremity
4. Hemihypertrophy of the right lower extremity

PREFERRED RESPONSE: 3

2013 American Academy of Orthopaedic Surgeons 2013 Pediatric OrthopaedicSelf-Assessment Examination


2013 Pediatric Orthopaedic Self-Assessment Examination Answer Book 43

DISCUSSION
Smallness of the entire lower extremity is intrinsic to clubfoot deformity. Multiple etiologies for size
difference have been investigated to include vascular, neurologic, and difference in tissue type. Arterial
abnormalities in clubfeet appear to produce a deletion phenomenon. Arteriography has shown that a
larger percentage of clubfeet have abnormal arterial patterns, with the posterior tibial artery being the
predominant vascular supply. Studies suggest that the absence of peripheral nerves or denervation
decreases the rate of bone maturation, and this effect may be responsible for the diminished size of
denervated limbs and may explain this phenomenon in clubfoot. However, bone age has been found to be
similar in those with clubfoot and controls. Tethering of the cord has not been found to be associated with
idiopathic clubfoot. The majority of clubfeet have relatively normal muscle fibers. Hemihypertrophy of
the contralateral limb is not associated with clubfoot deformity.

RECOMMENDED READINGS
Dietz FR. Effect of denervation on limb growth. J Orthop Res. 1989;7(2):292-303. PubMed PMID:
2783965.
Dietz FR. Effect of peripheral nerve on limb development. J Orthop Res. 1987;5(4):576-85. PubMed
PMID: 3500294.
Greider TD, Siff SJ, Gerson P, Donovan MM. Arteriography in club foot. J Bone Joint Surg Am. 1982
Jul;64(6):837-40. PubMed PMID: 7085711.
Herceg MB, Weiner DS, Agamanolis DP, Hawk D. Histologic and histochemical analysis of muscle
specimens in idiopathic talipes equinovarus. J Pediatr Orthop. 2006 Jan-Feb;26(1):91-3. PubMed PMID:
16439910.
Hootnick DR, Packard DS Jr, Levinsohn EM, Crider RJ Jr. Confirmation of arterial deficiencies in a limb
with necrosis following clubfoot surgery. J Pediatr Orthop B. 1999 Jul;8(3):187-93. PubMed PMID:
10399121.
Shimode K, Miyagi N, Majima T, Yasuda K, Minami A. Limb length and girth discrepancy of unilateral
congenital clubfeet. J Pediatr Orthop B. 2005 Jul;14(4):280-4. PubMed PMID: 15931033.
Spiegel DA, Loder RT. Leg-length discrepancy and bone age in unilateral idiopathic talipes equinovarus. J
Pediatr Orthop. 2003 Mar-Apr;23(2):246-50. PubMed PMID: 12604959.

2013 American Academy of Orthopaedic Surgeons 2013 Pediatric Orthopaedic Self-Assessment Examination
44 American Academy of Orthopaedic Surgeons

RESPONSES FOR QUESTIONS 38 THROUGH 43


1. Pavlik harness treatment
2. Immediate spica casting
3. Titanium elastic intramedullary nailing
4. Submuscular bridge plating
5. Antegrade femoral nailing with a trochanteric entry intramedullary nail

For each of the following fractures, select the most appropriate treatment from the list of responses
above.

Question 38
A 3-month-old infant with a displaced spiral midshaft femur fracture.

PREFERRED RESPONSE: 1

Question 39
A 16-year-old boy with a transverse midshaft femur fracture.

PREFERRED RESPONSE: 5

Question 40
A 2-year-old girl with a displaced spiral midshaft femur fracture.

PREFERRED RESPONSE: 2

Question 41
A 37-kg 9-year-old boy with a transverse midshaft femur fracture.

PREFERRED RESPONSE: 3

2013 American Academy of Orthopaedic Surgeons 2013 Pediatric OrthopaedicSelf-Assessment Examination


2013 Pediatric Orthopaedic Self-Assessment Examination Answer Book 45

Question 42
An 8-year-old boy with a severely comminuted and length-unstable midshaft femur fracture.

PREFERRED RESPONSE: 4

Question 43
A 9-year-old boy who was in a motor vehicle collision and sustained a head injury and grade 1 open
transverse midshaft femur fracture.

PREFERRED RESPONSE: 3

DISCUSSION FOR QUESTIONS 38 THROUGH 43


The treatment of pediatric femur fractures varies with the childs age and fracture characteristics. Children
from birth to age 6 months are treated with Pavlik harness application, which provides adequate pain
relief and alignment for the short duration required for healing in this age group. Between 6 months
and 5 years of age, most children are treated with immediate spica casting. While there may be some
exceptions (with heavier children undergoing titanium elastic nail fixation), this has been more a matter
of anecdote than study. While traction remains a treatment option for many femur fractures, many centers
have moved away from it because of the associated costs, both to payers and families, and the good
results achieved with other methods. Because traction remains a viable treatment option for many femur
fractures, it was left out of the suggested responses. Children age 5 to 11 with appropriate fractures are
by and large treated with titanium elastic intramedullary nails. One exception is children with severely
comminuted and length-unstable femur fracture, for whom titanium elastic nails may be less reliable and
submuscular bridge plating is an excellent choice. Titanium elastic nails have also been used without
undue complications in the treatment of low-grade open femur fractures, and may offer advantages over
external fixation such as improved wound access and decreased refracture rates. For children older than
age 11, rigid antegrade nailing through a trochanteric entry point is common. The use of a piriformis entry
nail is not recommended in children.

RECOMMENDED READINGS FOR QUESTIONS 38 THROUGH 43


Flynn JM, Schwend RM. Management of pediatric femoral shaft fractures. J Am Acad Orthop Surg. 2004
Sep-Oct;12(5):347-59. Review. PubMed PMID: 15469229.
Kocher MS, Sink EL, Blasier RD, Luhmann SJ, Mehlman CT, Scher DM, Matheney T, Sanders JO,
Watters WC III, Goldberg MJ, Keith MW, Haralson RH III, Turkelson CM, Wies JL, Sluka P, Hitchcock
K. Treatment of pediatric diaphyseal femur fractures. J Am Acad Orthop Surg. 2009 Nov;17(11):718-25.
Review. PubMed PMID: 19880682
Hedequist DJ, Sink E. Technical aspects of bridge plating for pediatric femur fractures. J Orthop Trauma.
2005 Apr;19(4):276-9. PubMed PMID: 15795577.
Kay RM, Skaggs DL. Pediatric polytrauma management. J Pediatr Orthop. 2006 Mar-Apr;26(2):268-77.
PubMed PMID: 16557148.

END OF SERIES

2013 American Academy of Orthopaedic Surgeons 2013 Pediatric Orthopaedic Self-Assessment Examination
46 American Academy of Orthopaedic Surgeons

Question 44
A 3-year-old girl was evaluated for toe walking. Her history was remarkable for prematurity with
known intracranial hemorrhage. Her recent neurologic workup included a brain MRI scan showing
periventricular leukomalacia. She walked at age 2. Gait examination revealed a crouched gait on tiptoes
with knees and hips flexed. Passive range-of-motion testing revealed hip flexion contracture of 5 degrees,
hamstring contracture of 15 degrees on popliteal angle testing, and ankles dorsiflex to neutral in knee
extension and 15 degrees above neutral in knee flexion. Initial treatment should consist of

1. percutaneous Achilles lengthening.


2. open Achilles tendon lengthening.
3. ankle bracing and therapy.
4. steroid therapy.

PREFERRED RESPONSE: 3

DISCUSSION
Bracing and therapy are the best initial treatments. The history and examination reveal spastic diplegia
GMFCS-II grade, which is a static brain injury that is not progressive. Brain MRI scan findings of
periventricular leukomalacia are often seen in spastic diplegia. Her examination shows mild contractures
on range-of-motion testing. The ankle range of motion examination shows a gastrocnemius contracture;
dorsiflexion is much improved in knee flexion. With a true Achilles contracture, ankle dorsiflexion does
not improve in knee flexion. Her history does not support muscular dystrophy, for which steroid therapy
can improve short-term function. Surgery is not indicated at this age with this amount of contracture.

RECOMMENDED READINGS
Shore BJ, White N, Kerr Graham H. Surgical correction of equinus deformity in children with cerebral
palsy: a systematic review. J Child Orthop. 2010 Aug;4(4):277-90. Epub 2010 Jul 1. PubMed PMID:
21804889.
Renshaw TS, Deluca PA. Cerebral palsy. In: Morrissy RT, Weinstein SL, eds. Lovell and Winters
Pediatric Orthopaedics. 6th ed. Philadelphia, PA: Lippincott Williams & Wilkins; 2006:551-604.

Question 45
What is the minimum hours per day of wear that has been correlated with the effectiveness of bracing on
curve progression in idiopathic scoliosis?

1. Prescribed brace wear 23 hours/day


2. Prescribed brace wear 16 hours/day
3. Actual brace wear more than 12 hours/day
4. Actual brace wear 6 hours/day

PREFERRED RESPONSE: 3

2013 American Academy of Orthopaedic Surgeons 2013 Pediatric OrthopaedicSelf-Assessment Examination


2013 Pediatric Orthopaedic Self-Assessment Examination Answer Book 47

DISCUSSION
The efficacy of brace treatment for patients with adolescent idiopathic scoliosis is controversial because its
effectiveness remains unproven. One of the challenges is patient noncompliance with prescribed bracing
regimens. A recent study investigated curve progression based on actual brace wear using a temperature
sensor to accurately assess brace wear. The total hours of brace wear correlated with lack of curve
progression with a dose-response effect noted. Curves did not progress in 82% of patients who actually
wore the brace more than 12 hours per day. For those who wore the brace for fewer than 7 hours per day,
curves progressed in 69%. Prescribed bracing regimens (eg, 16 hours/day or 23 hours/day) had no effect
on actual brace wear or curve progression.

RECOMMENDED READINGS
Katz DE, Herring JA, Browne RH, Kelly DM, Birch JG. Brace wear control of curve progression
in adolescent idiopathic scoliosis. J Bone Joint Surg Am. 2010 Jun;92(6):1343-52. PubMed PMID:
20516309.
Morton A, Riddle R, Buchanan R, Katz D, Birch J. Accuracy in the prediction and estimation of adherence
to bracewear before and during treatment of adolescent idiopathic scoliosis. J Pediatr Orthop. 2008 Apr-
May;28(3):336-41. PubMed PMID: 18362800.

Question 46
Children develop hand dominance by approximately what age?

1. 6 months to 9 months
2. 12 months to 18 months
3. 2 years to 3 years
4. 4 years to 5 years

PREFERRED RESPONSE: 3

DISCUSSION
Most children develop hand dominance during their third year. Although there is individual variation
in age of development of handedness, infants and toddlers are normally ambidextrous. Strong hand
preference in a child younger than age 2 may indicate a neurological deficit. Persistence of ambidexterity
is not pathological.

RECOMMENDED READINGS
Feigelman S. The preschool years. In: Kliegman RM, ed. Nelson Textbook of Pediatrics. 19th ed.
Philadelphia, PA: Saunders; 2011:chap 10.
Herring JA, ed. Growth and development. In: Tachdjians Pediatric Orthopaedics. 4th ed. Philadelphia,
PA: Saunders Elsevier; 2008:21-22.

2013 American Academy of Orthopaedic Surgeons 2013 Pediatric Orthopaedic Self-Assessment Examination
48 American Academy of Orthopaedic Surgeons

Figure 47

Question 47
What is the structure marked by the B on Figure 47?

1. Illium
2. Femoral head
3. Triradiate cartilage
4. Labrum

PREFERRED RESPONSE: 2

DISCUSSION
This is a coronal flexion view of a hip ultrasound. It is often best interpreted as an anteroposterior view
rotated 90 degrees to the left. The femoral head is the large circular structure (B) surrounded by the
acetabulum. The ilium (A) forms the roof. The triradiate cartilage(C) and ischium (D) form the medial
wall.

RECOMMENDED READINGS
Developmental dysplasia of the hip. In: Herring JA, ed. Tachdjians Pediatric Orthopaedics. 4th ed.
Philadelphia, PA: Saunders Elsevier; 2008:654-655.
Harcke HT, Kumar SJ. The role of ultrasound in the diagnosis and management of congenital dislocation
and dysplasia of the hip. J Bone Joint Surg Am. 1991 Apr;73(4):622-8. Review. PubMed PMID: 2013607.

2013 American Academy of Orthopaedic Surgeons 2013 Pediatric OrthopaedicSelf-Assessment Examination


2013 Pediatric Orthopaedic Self-Assessment Examination Answer Book 49

Figure 48
Question 48
Figure 48 is the radiograph of an 11-year-old boy who fell directly on his left shoulder while riding a
bicycle. He complained of pain in his left shoulder. An MRI scan of the injury is likely to reveal what
findings?

1. Disruption of the coracoclavicular ligaments


2. Dislocation of the acromioclavicular joint
3. Impingement of the inferior fragment on the brachial plexus
4. Intact periosteal sleeve inferiorly

PREFERRED RESPONSE: 4

DISCUSSION
In children, fractures of the distal clavicle are almost always through the distal physis and adjacent
metaphysis, and, consequently, the acromioclavicular joint is rarely dislocated. The coracoclavicular
ligaments usually remain attached to the thick periosteum on the undersurface of the clavicle and are
rarely damaged. Because of the thick periosteum and intact ligaments, these fractures are inherently stable
and heal well with conservative treatment. In contrast, similar injuries in adults pose a higher risk for
surgical intervention.

RECOMMENDED READINGS
Sarwark JF, King EC, Luhmann SJ. Proximal humerus, scapula, and clavicle. In: Beaty JH, Kasser JR,
eds. Rockwood and Wilkins Fractures in Children. 6th ed. Philadelphia, PA: Lippincott Williams &
Wilkins; 2006:chap 17.
Black GB, McPherson JA, Reed MH. Traumatic pseudodislocation of the acromioclavicular joint in
children. A fifteen year review. Am J Sports Med. 1991 Nov-Dec;19(6):644-6. PubMed PMID: 1781505.

2013 American Academy of Orthopaedic Surgeons 2013 Pediatric Orthopaedic Self-Assessment Examination
50 American Academy of Orthopaedic Surgeons

Figure 49a Figure 49b

Question 49
Figure 49a is the photograph of the axilla and Figure 49b is the radiograph of the spine of a 13-year-old
child referred for scoliosis treatment by a school screening program. What is the most likely diagnosis?

1. Idiopathic scoliosis
2. Nondystrophic scoliosis in neurofibromatosis I (NF1)
3. Nondystrophic scoliosis in NF2
4. Dystrophic scoliosis in NF2

PREFERRED RESPONSE: 2

DISCUSSION
Scoliosis is the most common skeletal manifestation of NF1. Nondystrophic scoliosis is less common
than dystrophic scoliosis. Its clinical appearance and behavior mimic idiopathic scoliosis. Dystrophic
scoliosis is recognized earlier than nondystrophic. It is characterized by a sharp angular curve involving 4
to 6 vertebrae. Its clinical appearance and behavior mimic idiopathic scoliosis. Scoliosis is not found in
NF2. Axillary freckling is common in NF1 or NF2.

RECOMMENDED READINGS
Crawford AH, Schorry EK. Neurofibromatosis in children: the role of the orthopaedist. J Am Acad Orthop
Surg. 1999 Jul-Aug;7(4):217-30. Review. PubMed PMID: 10434076.
Herring JA, ed. Tachdjians Pediatric Orthopaedics. 4th ed. Philadelphia, PA: Saunders Elsevier;
2008:376-379.

2013 American Academy of Orthopaedic Surgeons 2013 Pediatric OrthopaedicSelf-Assessment Examination


2013 Pediatric Orthopaedic Self-Assessment Examination Answer Book 51

Question 50
The AP radiograph of a 5-year-old boy shows a 20-degree left thoracic scoliosis. He was noted by his
pediatrician to have asymmetry on a forward bend test. On examination he is neurologically intact except
for decreased sensation on the lateral aspect of both flanks and to pinprick in both hands. He has no pain.
What is the best initial step in treatment at this time?

1. Observation
2. Initiate bracing
3. MRI scan of the entire spine
4. Spinal instrumentation with growing rod construct

PREFERRED RESPONSE: 3

DISCUSSION
This patient has atypical scoliosis, given his young age and left thoracic curve. In addition, he has
abnormal neurologic findings. MRI scan to evaluate for neural axis abnormalities is indicated. Abnormal
MRI findings are present in 2% to 3.8% of all patients with presumed idiopathic scoliosis. Abnormal MRI
findings are more likely if specific clinical factors are present, such as absence of thoracic apical segment
lordosis, atypical curve pattern, an abnormal neurologic examination, male gender, and age younger than
11. In a patient with an atypical curve and neurologic indicators, the yield of MRI scan for a neuraxis
abnormality has been shown to be 25%. This patient had both syringomyelia and a Chiari malformation
that were treated neurosurgically. Observation would have missed these findings. Bracing or spinal
instrumentation may eventually be treatment options for scoliosis given his young age, but establishing a
diagnosis first with an MRI scan of the spine is the most appropriate initial step.

RECOMMENDED READINGS
Davids JR, Chamberlin E, Blackhurst DW. Indications for magnetic resonance imaging in presumed
adolescent idiopathic scoliosis. J Bone Joint Surg Am. 2004 Oct;86-A(10):2187-95. PubMed PMID:
15466727.
Nakahara D, Yonezawa I, Kobanawa K, Sakoda J, Nojiri H, Kamano S, Okuda T, Kurosawa H. Magnetic
resonance imaging evaluation of patients with idiopathic scoliosis: a prospective study of four hundred
seventy-two outpatients. Spine (Phila Pa 1976). 2011 Apr 1;36(7):E482-5. PubMed PMID: 20479697.

2013 American Academy of Orthopaedic Surgeons 2013 Pediatric Orthopaedic Self-Assessment Examination
52 American Academy of Orthopaedic Surgeons

Figure 51a Figure 51b

CLINICAL SITUATION FOR QUESTIONS 51 AND 52


Figures 51a and 51b are the radiographs of a 12-year-old girl who fell off her bicycle.

Question 51
What is the most appropriate initial treatment?

1. Closed reduction and spica cast


2. Anatomic reduction, compression fixation, and spica immobilization
3. Crutches and nonweight-bearing activity for 6 weeks
4. Percutaneous Kirschner wire fixation

PREFERRED RESPONSE: 2

Question 52
What is the incidence of osteonecrosis in the injury shown in Figures 51a and 51b?

1. 25%
2. 50%
3. 75%
4. 100%

PREFERRED RESPONSE: 2

DISCUSSION
Fractured hips in a child demand anatomic reduction with compression fixation. This type of fracture
is mildly displaced and can usually be closed, reduced, and pinned. A hip spica cast is recommended to
prevent loss of reduction. Open reduction is recommended in a displaced fracture that cannot be reduced.
Kirschner wire fixation is not adequate. Osteonecrosis is the most common complication of hip fractures
in children. It occurs in 50% of transcervical fractures similar to the one shown in the radiographs. It also
occurs in 100% of transepiphyseal fractures, 25% of cervico-trochanteric fractures, and 10% to 15% of
peritrochanteric fractures.
2013 American Academy of Orthopaedic Surgeons 2013 Pediatric OrthopaedicSelf-Assessment Examination
2013 Pediatric Orthopaedic Self-Assessment Examination Answer Book 53

RECOMMENDED READINGS
Herring JA, ed. Tachdjians Pediatric Orthopaedics. 4th ed. Philadelphia; PA: Saunders Elsevier;
2008:2609-2619.
Canale ST, Beaty JH. Pelvic and hip fractures. In: Rockwood CA, Wilkins KE, Beaty JH, eds. Rockwood
and Greens Fractures in Children. 4th ed. Philadelphia, PA: Lippincott-Raven; 1996:1147-1175.

END OF SERIES

Figure 53

Question 53
Figure 53 is the anteroposterior radiograph of a childs fractured tibia. The parents want to know why the
bone is abnormal. Why is the bone thicker?

1. Increased bone remodeling


2. Decreased hydroxyapatite
3. Decrease in the number of osteoblasts
4. Decrease in the acidification of Howships lacuna

PREFERRED RESPONSE: 4

DISCUSSION
Albers-Schonberg disease or osteopetrosis is caused by an inability of the osteoclast to acidify Howships
lacuna. The lack of hydrochloric acid causes incomplete dissolution of hydroxyapatite and incomplete
remodeling. The bone remains disorganized and thick. This condition is not diagnosed in some patients
with osteopetrosis until a fracture occurs. Osteoblast activity is not decreased in the condition.

2013 American Academy of Orthopaedic Surgeons 2013 Pediatric Orthopaedic Self-Assessment Examination
54 American Academy of Orthopaedic Surgeons

RECOMMENDED READINGS
Landa J, Margolis N, Di Cesare P. Orthopaedic management of the patient with osteopetrosis. J Am Acad
Orthop Surg. 2007 Nov;15(11):654-62. Review. PubMed PMID: 17989416.
Armstrong DG, Newfield JT, Gillespie R. Orthopedic management of osteopetrosis: results of a survey
and review of the literature. J Pediatr Orthop. 1999 Jan-Feb;19(1):122-32. Review. PubMed PMID:
9890301.

Figure 54

Question 54
Figure 54 is the lateral radiograph of an 8-month-old infant whose mother reports he has not been moving
his left leg for 2 to 3 days. His mother denies any known trauma, and says he has not been febrile or
ill-appearing. The mother says her infant spends a significant amount of time being cared for by her
boyfriend and attending a daycare facility. What is the most likely cause of his injury?

1. Vitamin D deficiency
2. Nonaccidental trauma
3. Osteogenesis imperfecta
4. Osteomyelitis

PREFERRED RESPONSE: 2

DISCUSSION
The radiograph reveals a supracondylar femur fracture in an infant. There is evidence of early fracture
healing. All femur fractures in children under age 3 should receive a higher index of suspicion for
nonaccidental trauma. Considering that this injury was not witnessed and the child has been cared for
by multiple parties other than the mother who brought him to the emergency department, an immediate
consultation with child protective services is appropriate. There is no radiographic evidence of rickets, as
would be seen in vitamin D deficiency. Fractures in patients with osteogenesis imperfecta are typically
diaphyseal. There is no evidence of osteomyelitis.

2013 American Academy of Orthopaedic Surgeons 2013 Pediatric OrthopaedicSelf-Assessment Examination


2013 Pediatric Orthopaedic Self-Assessment Examination Answer Book 55

RECOMMENDED READINGS
Hui C, Joughin E, Goldstein S, Cooper N, Harder J, Kiefer G, Parsons D, Howard J. Femoral fractures
in children younger than three years: the role of nonaccidental injury. J Pediatr Orthop. 2008 Apr-
May;28(3):297-302. PubMed PMID: 18362793.
Baldwin K, Pandya NK, Wolfgruber H, Drummond DS, Hosalkar HS. Femur fractures in the pediatric
population: abuse or accidental trauma? Clin Orthop Relat Res. 2011 Mar;469(3):798-804. PubMed
PMID: 20373153.

Figure 55a Figure 55b

Question 55
Figures 55a and 55b are the radiographs of a 5-year-old boy 6 weeks after he was treated for a minimally
displaced tibia fracture in a long-leg cast that was recently removed by his pediatrician. He has been
afebrile and laboratory evaluation reveals a white blood cell count of 8.3 x109/L (reference range, 4.5-11.0
x109/L), C-reactive protein of < 0.5 (reference range, 0.08-3.1 mg/L), and erythrocyte sedimentation rate
of 2 mm/h (reference range, 0-20 mm/h). In addition to axillary freckling, what physical examination
finding is most likely to be present in this patient?

1. Posteromedial bowing of his contralateral tibia


2. Multiple caf-au-lait spots on his trunk and extremities
3. Calcaneal valgus deformity of the foot
4. Visual disturbance attributable to glaucoma

PREFERRED RESPONSE: 2

2013 American Academy of Orthopaedic Surgeons 2013 Pediatric Orthopaedic Self-Assessment Examination
56 American Academy of Orthopaedic Surgeons

DISCUSSION
Congenital pseudoarthrosis is a rare condition occasionally found in patients with neurofibromatosis
(NF). The diagnosis can sometimes be made with progressive anterolateral bowing of the tibia, but often
presents as a fracture through the residual dysplastic tibia that presents with nonhealing. Patients with
NF1 commonly have axillary and groin freckling, multiple caf-au-lait spots, optic gliomas, and Lisch
nodules. Other orthopaedic manifestations include scoliosis, cervical spine abnormalities, and other bone
dysplasias or pseudoarthrosis (clavicle, radius). NF and congenital pseudoarthrosis are associated with
anterolateral bowing of the tibia, not posteromedial bowing, which can be associated with calcaneal valgus
deformity of the foot. In addition, NF is associated with optic glioma visual disturbances, not glaucoma.

RECOMMENDED READINGS
Feldman DS, Jordan C, Fonseca L. Orthopaedic manifestations of neurofibromatosis type 1. J Am Acad
Orthop Surg. 2010 Jun;18(6):346-57. Review. PubMed PMID: 20511440.
Vander Have KL, Hensinger RN, Caird M, Johnston C, Farley FA. Congenital pseudarthrosis of the tibia. J
Am Acad Orthop Surg. 2008 Apr;16(4):228-36. Review. PubMed PMID: 18390485.

Figure 56a Figure 56b

Question 56
Figures 56a and 56b are the MRI scans of a 2-year-old girl who has a fever of 39C and inability to move
her left arm. She has not had any recent injury and is otherwise healthy. Radiograph findings of her left
upper extremity are normal. What is the most appropriate treatment?

1. Incision and drainage of the humerus


2. Sling immobilization of the left upper extremity
3. Total body bone scan
4. A course of oral antibiotics and recheck in 1 week

PREFERRED RESPONSE: 1

2013 American Academy of Orthopaedic Surgeons 2013 Pediatric OrthopaedicSelf-Assessment Examination


2013 Pediatric Orthopaedic Self-Assessment Examination Answer Book 57

DISCUSSION
The MRI scan shows a large abscess along most of the humerus. Incision and drainage of the abscess
is indicated. A bone scan will not add any additional information regarding location of the problem.
Immobilization will not address the origin of the problem. Oral antibiotics are not adequate for this
abscess.

RECOMMENDED READINGS
Jones HW, Beckles VL, Akinola B, Stevenson AJ, Harrison WJ. Chronic haematogenous osteomyelitis in
children: an unsolved problem. J Bone Joint Surg Br. 2011 Aug;93(8):1005-10. Review. PubMed PMID:
21768620.
Conrad DA. Acute hematogenous osteomyelitis. Pediatr Rev. 2010 Nov;31(11):464-71. Review. PubMed
PMID: 21041424.

Figure 57

Question 57
Figure 57 is the lateral radiograph of a 14-year-old obese boy who sustained an injury to his right knee
while playing soccer. He has a large effusion and pain with any motion. The decision is made to perform
surgery. What structure is most likely to be interposed in the fracture site?

1. Medial meniscus
2. Intermeniscal ligament
3. Lateral meniscus
4. Osteochondral fragment

PREFERRED RESPONSE: 1

2013 American Academy of Orthopaedic Surgeons 2013 Pediatric Orthopaedic Self-Assessment Examination
58 American Academy of Orthopaedic Surgeons

DISCUSSION
Records of a consecutive series of 80 skeletally immature patients (mean age, 11.6 years; range, ages 5
to 16) who underwent arthroscopic (71), open (5), or combined arthroscopic and open (4) reduction and
internal fixation of type 3 tibial eminence fractures (57) or type 2 fractures that did not reduce in extension
(23) between 1993 and 2001 were reviewed. Entrapment of the anterior horn of the medial meniscus
(36), intermeniscal ligament (6), or anterior horn of the lateral meniscus (1) was seen in 26% (6 of 23) of
type 2 fractures and 65% (37 of 57) of type 3 fractures. An associated meniscal tear was seen in 3.8% of
patients (3 of 80). Meniscal entrapment is common in patients with type 2 and 3 tibial eminence fractures.
Arthroscopic or open reduction should be considered for type 3 fractures and for type 2 fractures that do
not reduce in extension to remove the incarcerated meniscus, allowing for anatomic reduction.

RECOMMENDED READINGS
Kocher MS, Micheli LJ, Gerbino P, Hresko MT. Tibial eminence fractures in children: prevalence of
meniscal entrapment. Am J Sports Med. 2003 May-Jun;31(3):404-7. PubMed PMID: 12750134.
Chandler JT, Miller TK. Tibial eminence fracture with meniscal entrapment. Arthroscopy. 1995
Aug;11(4):499-502. PubMed PMID: 7575887.

Figure 58

Question 58
Figure 58 is the radiograph of a 5-year-old boy who has left thigh pain. What is the most appropriate
treatment?

1. Valgus osteotomy with internal fixation


2. Curettage and allograft
3. Observation
4. Spica casting

PREFERRED RESPONSE: 1

2013 American Academy of Orthopaedic Surgeons 2013 Pediatric OrthopaedicSelf-Assessment Examination


2013 Pediatric Orthopaedic Self-Assessment Examination Answer Book 59

DISCUSSION
This child has symptomatic fibrous dysplasia as evidenced by the coxa vara and ground-glass appearance.
Lucency is also seen more distally in the femoral shaft. He already is symptomatic and is likely to
progress to more severe deformity with observation alone. Although surgery may need to be repeated,
corrective osteotomy and fixation are most appropriate. Allograft bone is likely to be resorbed in fibrous
dysplasia. Casting is unlikely to relieve symptoms for long. Biopsy can be performed in conjunction with
surgical correction.

RECOMMENDED READINGS
DiCaprio MR, Enneking WF. Fibrous dysplasia. Pathophysiology, evaluation, and treatment. J Bone Joint
Surg Am. 2005 Aug;87(8):1848-64. Review. PubMed PMID: 16085630.
Parekh SG, Donthineni-Rao R, Ricchetti E, Lackman RD. Fibrous dysplasia. J Am Acad Orthop Surg.
2004 Sep-Oct;12(5):305-13. Review. PubMed PMID: 15469225.

Figure 59a Figure 59b

Question 59
Figures 59a and 59b are the radiographs of a 7-year-old boy who was seen 1 week after he underwent a
closed reduction and casting in the emergency department after a fall on an outstretched arm. What is the
most appropriate next step for this patient?

1. Observation
2. Repeat closed reduction and casting
3. Open reduction and plate fixation
4. Closed reduction and intramedullary nail fixation

PREFERRED RESPONSE: 1

2013 American Academy of Orthopaedic Surgeons 2013 Pediatric Orthopaedic Self-Assessment Examination
60 American Academy of Orthopaedic Surgeons

DISCUSSION
This childs radiograph shows an acceptably reduced fracture of both the radius and ulna. Generally
accepted limits of shaft angulation for cast treatment for girls 8 years of age or younger and boys age
10 or younger are 20 degrees for distal-third, 15 degrees for middle-third, and 10 degrees for proximal-
third fractures. Remodeling decreases as one goes from distal to proximal in the forearm. Unless the
childs fracture deviates from these criteria, surgical treatment is not necessary. Because of the risk of
displacement, however, close follow-up is recommended.

RECOMMENDED READINGS
Bowman EN, Mehlman CT, Lindsell CJ, Tamai J. Nonoperative treatment of both-bone forearm shaft
fractures in children: predictors of early radiographic failure. J Pediatr Orthop. 2011 Jan-Feb;31(1):23-32.
PubMed PMID: 21150728.
Noonan KJ, Price CT. Forearm and distal radius fractures in children. J Am Acad Orthop Surg. 1998 May-
Jun;6(3):146-56. Review. PubMed PMID: 9689186.

Question 60
A 3-year-old boy who was treated with Ponseti-method casting demonstrates supination of the affected
foot during swing phase of gait. His ankle has 20 degrees of active and passive dorsiflexion and 45
degrees of plantar flexion. What is the most appropriate treatment?

1. Anterior tibialis tendon lengthening


2. Anterior tibialis tendon transfer to a dorsal lateral foot position
3. Anterior tibialis tendon transfer through the interosseous membrane to the calcaneus
4. Posterior tibialis tendon transfer through the interosseous membrane to the dorsum of the foot

PREFERRED RESPONSE: 2

DISCUSSION
Dynamic supination is a known residual problem after the Ponseti casting technique is used. Ponseti
applied the principles of the Garceau procedure and transferred the entire anterior tibialis tendon to
the lateral cuneiform. The anterior tibialis continues to act as a dorsiflexor of the foot but with a more
advantageous trajectory. Lengthening the anterior tibialis by itself will not redirect the pull of the muscle.
Likewise, the posterior tibialis tendon transfer through the interosseous membrane will not address the
supinating force of the anterior tibialis. It is used when the anterior and lateral compartment muscles are
either not functioning or weak in relationship to the posterior tibialis. The anterior tibialis tendon transfer
to the calcaneus has been used successfully in myelodysplasia for calcaneal deformity.

2013 American Academy of Orthopaedic Surgeons 2013 Pediatric OrthopaedicSelf-Assessment Examination


2013 Pediatric Orthopaedic Self-Assessment Examination Answer Book 61

RECOMMENDED READINGS
Bliss DG, Menelaus MB. The results of transfer of the tibialis anterior to the heel in patients who have a
myelomeningocele. J Bone Joint Surg Am. 1986 Oct;68(8):1258-64. PubMed PMID: 3771607.
Ezra E, Hayek S, Gilai AN, Khermosh O, Wientroub S. Tibialis anterior tendon transfer for residual
dynamic supination deformity in treated club feet. J Pediatr Orthop B. 2000 Jun;9(3):207-11. PubMed
PMID: 10904909.
Georgiadis GM, Aronson DD. Posterior transfer of the anterior tibial tendon in children who have a
myelomeningocele. J Bone Joint Surg Am. 1990 Mar;72(3):392-8. Erratum in: J Bone Joint Surg Am 1990
Jun;72(5):792. PubMed PMID: 2312535.
Garceau GJ, Manning KR. Transposition of the anterior tibial tendon in the treatment of recurrent
congenital club-foot. J Bone Joint Surg Am. 1947 Oct;29(4):1044-8. PubMed PMID: 20270370.
Ponseti IV, Smoley EN. Congenital club foot: the results of treatment. J Bone Joint Surg Am 1963;45:261-
275.

Question 61
The majority of cases of osteogenesis imperfecta (OI) are caused by mutations in genes that encode

1. fibrillin.
2. type I collagen.
3. type II collagen.
4. fibroblast growth factor receptor (FGFR)-3.

PREFERRED RESPONSE: 2

DISCUSSION
OI is caused by mutations in type I collagen genes COL1A1 or COL1A2 in 85% of cases; the remainder
of mutations are in collagen-associated proteins that destabilize the organic bone matrix. Marfan
syndrome is caused by mutations in fibrillin. A variety of epiphyseal dysplasias such as multiple
epiphyseal dysplasia have type II collagen mutations. Achondroplasia is caused by a mutation in FGFR-3.

RECOMMENDED READINGS
Sponseller PD, Ain MC. The skeletal dysplasias. In: Morrissy RT, Weinstein SL, eds. Lovell and Winters
Pediatric Orthopaedics. 6th ed. Philadelphia, PA: Lippincott Williams & Wilkins; 2006:205-250.
Shirley ED, Ain MC. Achondroplasia: manifestations and treatment. J Am Acad Orthop Surg. 2009
Apr;17(4):231-41. Review. PubMed PMID: 19307672.
Zeitlin L, Fassier F, Glorieux FH. Modern approach to children with osteogenesis imperfecta. J Pediatr
Orthop B. 2003 Mar;12(2):77-87. Review. PubMed PMID: 12584489.

2013 American Academy of Orthopaedic Surgeons 2013 Pediatric Orthopaedic Self-Assessment Examination
62 American Academy of Orthopaedic Surgeons

Question 62
What genetic defect is responsible for achondroplasia?

1. Trisomy of chromosome 21
2. Defect in collagen, type 1, alpha 1 (COL1A1)
3. Defect in fibroblast growth factor 3 (FGF-3)
4. Defect in fibroblast growth factor receptor 3 (FGFR-3)

PREFERRED RESPONSE: 4

DISCUSSION
In achondroplasia the defect is always in the same location on this gene (a defect in FGFR-3), and most
children share a very similar clinical phenotype. Achondroplasia is not caused by a defect in the gene for
FGF-3, the growth factor itself, but rather the gene coding for the receptor. Trisomy of chromosome 21 is
responsible for Down syndrome. Defects in the COL1A1 gene are found in some types of osteogenesis
imperfecta. Unlike achondroplasia, the defects occur throughout the gene, with more than 200 mutation
sites reported.

RECOMMENDED READINGS
Shirley ED, Ain MC. Achondroplasia: manifestations and treatment. J Am Acad Orthop Surg. 2009
Apr;17(4):231-41. Review. PubMed PMID: 19307672.
Karol LA. Skeletal Dysplasias. In: Herring JA, ed. Tachdjians Pediatric Orthopaedics. Vol 2. 4th ed.
Philadelphia, PA: Saunders Elsevier; 2008:1683-1695.

2013 American Academy of Orthopaedic Surgeons 2013 Pediatric OrthopaedicSelf-Assessment Examination


2013 Pediatric Orthopaedic Self-Assessment Examination Answer Book 63

Figure 63

Question 63
Figure 63 is the anteroposterior (AP) radiograph of a 12-year-old boy with type 3 spinal muscular atrophy
(SMA) whose scoliosis has progressed despite bracing. Because of his curvature, it has become more
difficult for him to support his trunk while sitting, and for the last 6 months he has required assisted
ventilation at night. What is the most appropriate treatment?

1. Growing rod construct


2. Anterior spinal fusion with instrumentation
3. Posterior spinal fusion with instrumentation
4. AP spinal fusion with instrumentation

PREFERRED RESPONSE: 3

DISCUSSION
Surgery is recommended for scoliosis associated with SMA when curves progress beyond 50 to 60 degrees
and are associated with impaired sitting balance and worsening pulmonary function. Continued bracing
is unlikely to be effective. Anterior surgery alone is not recommended because it is associated with
high rates of secondary curve progression in patients with SMA, and also may further impair pulmonary
function. Because curves in SMA are relatively flexible, combined anterior and posterior surgery is
usually required only for very large curves (more than 100 degrees) and very young patients (to prevent
crankshaft phenomenon), and should be reserved for those without significant pulmonary issues because
of the pulmonary morbidity associated with an anterior procedure. In this case, for an older child with a
60-degree curve and developing pulmonary difficulties, posterior surgery alone is the best option.

RECOMMENDED READINGS
Sucato DJ. Spine deformity in spinal muscular atrophy. J Bone Joint Surg Am. 2007 Feb;89 Suppl 1:148-
54. Review. Erratum in: J Bone Joint Surg Am. 2007 May;89(5):1090-1. PubMed PMID: 17272431.
Aprin H, Bowen JR, MacEwen GD, Hall JE. Spine fusion in patients with spinal muscular atrophy. J Bone
Joint Surg Am. 1982 Oct;64(8):1179-87. PubMed PMID: 7130230.
Sussman MD. Progressive neuromuscular diseases. In: Abel MF, ed. Orthopaedic Knowledge Update:
Pediatrics 3. Rosemont, IL: American Academy of Orthopaedic Surgeons; 2006:123-135.

2013 American Academy of Orthopaedic Surgeons 2013 Pediatric Orthopaedic Self-Assessment Examination
64 American Academy of Orthopaedic Surgeons

Question 64
A 2-year-old boy was examined for flatfoot when standing. Gait examination revealed normal toddler
gait with flat feet; in stance no arch is visible and heels are in 10 degrees of valgus. He has no pain. His
arches reconstitute when seated or on tiptoe. Passive ankle dorsiflexion is 15 degrees in knee extension,
plantar flexion is 50 degrees, and subtalar motion is 30 degrees. What is the most appropriate treatment?

1. Arch supports should be prescribed to help his feet develop.


2. Radiographs are needed to evaluate for deformities such as vertical talus or tarsal coalition.
3. Observation and education
4. Physical therapy should be prescribed to help his feet develop and stretch his tight Achilles
tendons.

PREFERRED RESPONSE: 3

DISCUSSION
Flexible flat feet are a normal toddler variant. In the absence of pain or dysfunction, flat feet do not need
treatment of any kind. Examination shows normal ankle motion, so Achilles stretching is not needed.
His range-of-motion and gait examination does not suggest any abnormalities; therefore, radiographs are
unnecessary.

RECOMMENDED READINGS
Mosca VS. Flexible flatfoot in children and adolescents. J Child Orthop. 2010 Apr;4(2):107-21. Epub
2010 Feb 18. PubMed PMID: 21455468.
Kasser JR. The foot. In: Morrissy RT, Weinstein SL, eds. Lovell and Winters Pediatric Orthopaedics. 6th
ed. Philadelphia, PA: Lippincott Williams & Wilkins; 2006:1257-1328.

2013 American Academy of Orthopaedic Surgeons 2013 Pediatric OrthopaedicSelf-Assessment Examination


2013 Pediatric Orthopaedic Self-Assessment Examination Answer Book 65

Figure 65a Figure 65b Figure 65c

Question 65
Figures 65a and 65b are the initial radiographs of a 13-year-old boy who sustained an injury to his right
knee while playing tackle football. He had immediate pain and the inability to bear weight. Open
treatment of the injury was recommended and his postoperative radiograph is shown in Figure 65c. He
and his family should be advised that his risk for distal femoral physeal arrest is approximately

1. 10%.
2. 30%.
3. 60%.
4. 95%.

PREFERRED RESPONSE: 3

DISCUSSION
Multiple studies have shown a fairly high rate of physeal arrest following physeal injuries to the distal
femur. Although individual studies list varying rates for physeal arrest, the overall rate is about 60%. The
rate has been shown to be higher when the initial fracture is displaced.

RECOMMENDED READINGS
Basener CJ, Mehlman CT, DiPasquale TG. Growth disturbance after distal femoral growth plate fractures
in children: a meta-analysis. J Orthop Trauma. 2009 Oct;23(9):663-7. PubMed PMID: 19897989.
Price CT, Flynn JM. Management of fractures. In: Morrissy RT, Weinstein ST, eds. Lovell and Winters
Pediatric Orthopaedics. 6th ed. Philadelphia, PA: Lippincott Williams & Wilkins; 2006:1495-1496.

2013 American Academy of Orthopaedic Surgeons 2013 Pediatric Orthopaedic Self-Assessment Examination
66 American Academy of Orthopaedic Surgeons

Question 66
A 5-week-old infant was evaluated for a unilateral clubfoot. The parents are told their child will have
Ponseti-type management of clubfoot. What is the best description of the initial phase of this treatment?

1. Weekly long-leg casts will be applied to first correct the cavus, adductus, and hindfoot varus
components, and then a percutaneous Achilles tenotomy will be used to correct the equinus
followed by a final cast.
2. Weekly short-leg casts will be applied to first correct the cavus, adductus, and hindfoot varus
components, and then a brace will be used to correct the equinus.
3. Weekly long-leg casts will be applied to correct the equinus first, then an abduction brace will
be used to correct the rest of the foot.
4. Weekly long-leg casts will be applied, abducting the forefoot by pushing against the
calcaneocuboid joint until the foot is straight, and then the casts will focus on the equinus.

PREFERRED RESPONSE: 1

DISCUSSION
The Ponseti technique consists of weekly long-leg groin-to-toe casts that correct the components of the
clubfoot in the order of cavus, forefoot adductus, hindfoot varus, and equinus. A percutaneous Achilles
tenotomy is needed to finish correcting the equinus in almost all cases because bracing a foot in equinus
will lead to brace failure attributable to the foot sliding out or blistering. The Ponseti technique entails
pushing against the laterally positioned talar head and reducing the foot around this fulcrum to the proper
position of the talonavicular joint. The technique described in Response 3 is Kites method, which created
a rocker-bottom foot. Short-leg casts are not used in the Ponseti method.

RECOMMENDED READINGS
Ponseti IV. Congenital Clubfoot: Fundamentals of Treatment. New York, NY: Oxford University Press;
1996.
Kasser JR. The foot. In: Morrissy RT, Weinstein ST, eds. Lovell and Winters Pediatric Orthopaedics. 6th
ed. Philadelphia, PA: Lippincott Williams & Wilkins; 2006:1257-1328.

Question 67
Laboratory examination of a 5-year-old boy who toe-walks and recently began tripping and falling
revealed a creatine phosphokinase level 10 times higher than the normal level. He has one 11-year-
old brother who walks normally. To confirm the diagnosis of Duchenne muscular dystrophy, a creatine
phosphokinase test should be ordered on which family member?

1. Father
2. Mother
3. Brother
4. Sister

PREFERRED RESPONSE: 2

2013 American Academy of Orthopaedic Surgeons 2013 Pediatric OrthopaedicSelf-Assessment Examination


2013 Pediatric Orthopaedic Self-Assessment Examination Answer Book 67

DISCUSSION
Creatine phosphokinase levels are elevated in this patient and the mother is the carrier. The fathers test
result is always negative because this is a sex-linked disease. The brothers test result will be normal
because he does not have the disease.

RECOMMENDED READINGS
Thompson GW, Benerenson FR. Sex linked muscular dystrophies. In: Morrissey RT, Weinstein SL, eds.
Lovell and Winters Pediatric Orthopaedics. 6th ed. Philadelphia, PA: Lippincott Williams and Wilkins;
2006: 653-660.
Muscle diseases. In: Herring JA, ed. Tachdjians Pediatric Orthopaedics. 4th ed. Philadelphia, PA:
Saunders Elsevier; 2008:1621-1653.

Video 68

Question 68
Video 68 shows a cast being removed from a childs forearm. The risk of cast saw burn can be
decreased by

1. sedating the child.


2. using fiberglass cast material.
3. using an alternative saw technique.
4. using a saw without a vacuum.

PREFERRED RESPONSE: 3

DISCUSSION
Although the heat generated may be related to the thickness of any material, fiberglass tends to generate
more heat than plaster. A sedated patient cannot inform the practitioner of any discomfort during use of
the saw. During the cutting of the cast depicted in the video, the saw never leaves the cast material. This
constant contact tends to produce more heat than an intermittent up and down technique. The saw
vacuum has been shown to contribute to the cooling of the blade.

2013 American Academy of Orthopaedic Surgeons 2013 Pediatric Orthopaedic Self-Assessment Examination
68 American Academy of Orthopaedic Surgeons

RECOMMENDED READINGS
Shuler FD, Grisafi FN. Cast-saw burns: evaluation of skin, cast, and blade temperatures generated during
cast removal. J Bone Joint Surg Am. 2008 Dec;90(12):2626-30. PubMed PMID: 19047707.
Killian JT, White S, Lenning L. Cast-saw burns: comparison of technique versus material versus saws. J
Pediatr Orthop. 1999 Sep-Oct;19(5):683-7. PubMed PMID: 10488876.

Question 69
A 3-year-old girl with spastic right hemiparesis walks on her toes. Her right ankle can be dorsiflexed
passively to 5 degrees short of neutral position. Her left ankle dorsiflexes actively and passively to +15
degrees. Botulinum toxin injections were given to her right calf 1 month before this examination. What is
the most appropriate treatment at this time?

1. Immediate fitting of right and left ankle-foot orthoses


2. Immediate fitting of a right ankle-foot orthosis
3. Percutaneous heel cord lengthening, right
4. Serial casting of the right foot and ankle

PREFERRED RESPONSE: 4

DISCUSSION
The patient walks in equinus on the unaffected left side in response to the deformity on the affected
right side. Once the equinus is controlled on the hemiparetic side, the other side will be able to function
properly and will not require orthotic management. Because of the development of the musculotendinous
unit of the gastrosoleus complex, it is best to delay surgical intervention for equinus deformity until this
child is at least 4 years of age. It will be difficult to obtain a properly fitting ankle-foot orthosis with even
a mild fixed deformity. Serial casting with or without botulinum toxin injection is a useful technique to
obtain a neutral ankle position and facilitate orthotic management.

RECOMMENDED READINGS
Boyd RN, Pliatsios V, Starr R, Wolfe R, Graham HK. Biomechanical transformation of the gastroc-soleus
muscle with botulinum toxin A in children with cerebral palsy. Dev Med Child Neurol. 2000 Jan;42(1):32-
41. PubMed PMID: 10665973.
Cottalorda J, Gautheron V, Metton G, Charmet E, Chavrier Y. Toe-walking in children younger than
six years with cerebral palsy. The contribution of serial corrective casts. J Bone Joint Surg Br. 2000
May;82(4):541-4. PubMed PMID: 10855878.
Brouwer B, Davidson LK, Olney SJ. Serial casting in idiopathic toe-walkers and children with spastic
cerebral palsy. J Pediatr Orthop. 2000 Mar-Apr;20(2):221-5. PubMed PMID: 10739286.

2013 American Academy of Orthopaedic Surgeons 2013 Pediatric OrthopaedicSelf-Assessment Examination


2013 Pediatric Orthopaedic Self-Assessment Examination Answer Book 69

Question 70
What is the most likely cause of an acute femur fracture in a 5-month-old child?

1. Metabolic bone disease


2. Birth trauma
3. Accidental trauma
4. Nonaccidental trauma (abuse)

PREFERRED RESPONSE: 4

DISCUSSION
Although femur fractures are common in children, fractures in nonambulatory children carry a very high
specificity for abuse. Metabolic bone disease is less commonly a cause of femur fractures. Birth trauma,
from which femur fractures have been reported, would have been healed by 5 months of age. Accidental
trauma generally does not occur in nonambulatory children. Orthopaedic surgeons, like all physicians, are
mandated to report suspected cases of child abuse.

RECOMMENDED READINGS
Kocher MS, Kasser JR. Orthopaedic aspects of child abuse. J Am Acad Orthop Surg. 2000 Jan-
Feb;8(1):10-20. Review. PubMed PMID: 10666649.
Loder RT, Feinberg JR. Orthopaedic injuries in children with nonaccidental trauma: demographics and
incidence from the 2000 kids inpatient database. J Pediatr Orthop. 2007 Jun;27(4):421-6. Erratum in: J
Pediatr Orthop. 2008 Sep;28(6):699. PubMed PMID: 17513964.
Pandya NK, Baldwin K, Wolfgruber H, Christian CW, Drummond DS, Hosalkar HS. Child abuse
and orthopaedic injury patterns: analysis at a level I pediatric trauma center. J Pediatr Orthop. 2009
Sep;29(6):618-25. PubMed PMID: 19700994.

2013 American Academy of Orthopaedic Surgeons 2013 Pediatric Orthopaedic Self-Assessment Examination
70 American Academy of Orthopaedic Surgeons

Figure 71

Question 71
Figure 71 is the MRI scan of a 2-year-old girl who has been febrile for 1 week and has refused to bear
weight on her left lower extremity for 3 days. Her entire left lower extremity is markedly swollen.
Doppler ultrasound shows a deep venous thrombosis of the internal iliac vein. Her white blood cell
count is 19000/L (reference range, 4500-11000/L) and her C-reactive protein level is higher than 20
mg/L (reference range, 0.08-3.1 mg/L). If blood cultures yield positive results, what is the most likely
organism?

1. Methicillin-resistant Staphylococcus aureus


2. Salmonella typhii
3. Escherichia coli
4. Vancomycin-resistant Enterococcus

PREFERRED RESPONSE: 1

DISCUSSION
The clinical picture is one of infection and deep venous thrombosis. The MRI scan is consistent with
osteomyelitis. Deep venous thrombosis in association with musculoskeletal infection is more common in
osteomyelitis caused by methicillin-resistant Staphylococcus aureus. Presenting C-reactive protein levels
generally are higher than 6 mg/L and are higher than with other causative organisms. The presence of the
Panton-Valentine leukocidin gene encoded in strains of bacteria may explain the deep venous thrombosis.

RECOMMENDED READINGS
Hollmig ST, Copley LA, Browne RH, Grande LM, Wilson PL. Deep venous thrombosis associated with
osteomyelitis in children. J Bone Joint Surg Am. 2007 Jul;89(7):1517-23. PubMed PMID: 17606791.
Gonzalez BE, Teruya J, Mahoney DH Jr, Hulten KG, Edwards R, Lamberth LB, Hammerman WA, Mason
EO Jr, Kaplan SL. Venous thrombosis associated with staphylococcal osteomyelitis in children. Pediatrics.
2006 May;117(5):1673-9. PubMed PMID: 16651323.

2013 American Academy of Orthopaedic Surgeons 2013 Pediatric OrthopaedicSelf-Assessment Examination


2013 Pediatric Orthopaedic Self-Assessment Examination Answer Book 71

Question 72
What is the most common type of skeletal dysplasia?

1. Diastrophic dysplasia
2. Multiple epiphyseal dysplasia
3. Nail patella syndrome
4. Achondroplasia

PREFERRED RESPONSE: 4

DISCUSSION
Achondroplasia is the most common skeletal dysplasia, with an incidence of 1/30,000 live births. The
other choices are all more rare.

RECOMMENDED READINGS
Shirley ED, Ain MC. Achondroplasia: manifestations and treatment. J Am Acad Orthop Surg. 2009
Apr;17(4):231-41. Review. PubMed PMID: 19307672.
Sponseller PD, Ain MC. The skeletal dysplasias. In: Morrissy RT, Weinstein SL, eds. Lovell and Winters
Pediatric Orthopaedics. 6th ed. Philadelphia, PA: Lippincott Williams & Wilkins; 2006:205-250.

Question 73
A 13-year-old boy was evaluated for leg length difference. His pelvis balanced when a 1-inch (2.54 cm)
block was placed under his left foot. History revealed he had a left distal femur physeal fracture treated
with casting at age 10. Radiographs show normal limb alignment, but his left distal femoral physis is
closed and his left femur is 2.5 cm shorter than the right. All other physes are open. His bone age is equal
to his chronologic age. What surgical treatments will best equalize his discrepancy?

1. Right distal femoral and proximal tibia/fibula epiphysiodesis


2. Right distal femoral epiphysiodesis
3. Right proximal tibia/fibula epiphysiodesis
4. Left proximal tibia/fibula epiphysiodesis

PREFERRED RESPONSE: 1

2013 American Academy of Orthopaedic Surgeons 2013 Pediatric Orthopaedic Self-Assessment Examination
72 American Academy of Orthopaedic Surgeons

DISCUSSION
Because the left distal femoral physis is closed with a leg length difference already at 1 inch,
epiphysiodesis of both the right distal femur and proximal tibia/fibula is needed. The amount of correction
will be the amount of growth remaining in the left proximal tibia. Presuming the standard rates of growth
of 10 mm per year distal femur, 6 mm per year proximal tibia, and 4 mm per year distal tibia, this should
yield a correction of 6 mm x 3 years = 1.8 cm by skeletal maturity at age 16. This would leave the boy
with an acceptable discrepancy of 7 mm, well under 1 inch/2.54 cm. Closing only the right distal femoral
physis will leave the discrepancy unchanged at 1 inch/2.54 cm because no growth differential will exist.
Closing the right proximal tibia/fibular physis would mean the left knee will grow at 6 mm per year, but
the right will grow at 10 mm per year. The discrepancy would increase by 4 mm per year, or 1.2 cm
by skeletal maturity of age 16, leaving the boy 3.7 cm short on the left. Closing the left proximal tibia/
fibula physis would increase the discrepancy at the rate of 1.6 cm per year x 3 years because both the right
femoral and proximal tibia/fibular physes would be growing, leaving the boy 4.8 + 2.5 = 7.3 cm shorter on
the left leg.

RECOMMENDED READINGS
Moseley C. Leg-length discrepancy. In: Morrissy RT, Weintein SL, eds. Lovell and Winters Pediatric
Orthopaedics. 6th ed. Philadelphia, PA: Lippincott Williams & Wilkins; 2006:1213-1256.
Spencer SA, Widmann RF. Limb-length discrepancy and limb lengthening. In: Song KM, ed. Orthopaedic
Knowledge Update: Pediatrics 4. Rosemont, IL: American Academy of Orthopaedic Surgeons; 2011:219-
232.

Question 74
The main blood supply to the capital femoral epiphysis in a 10-year-old child is supplied from the

1. artery of the ligamentum teres.


2. epiphyseal branch of the lateral femoral circumflex artery.
3. posterosuperior and posteroinferior retinacular branches of the lateral femoral circumflex
artery.
4. posterosuperior and posteroinferior retinacular branches of the medial femoral circumflex
artery.

PREFERRED RESPONSE: 4

DISCUSSION
Before the age of 4 years, blood to the femoral head is supplied by the medial and lateral femoral
circumflex arteries as well as the artery of the ligamentum teres. After the age of 4, the blood supply
through the artery of the ligamentum teres diminishes. The lateral femoral circumflex system regresses,
and its flow into the physis and epiphysis diminishes significantly so the medial femoral circumflex artery
becomes the predominant blood supply to the metaphysis. The medial femoral circumflex artery provides
the principal blood supply to the proximal femur via its posterosuperior and posteroinferior retinacular
branches.

2013 American Academy of Orthopaedic Surgeons 2013 Pediatric OrthopaedicSelf-Assessment Examination


2013 Pediatric Orthopaedic Self-Assessment Examination Answer Book 73

RECOMMENDED READINGS
Boardman MJ, Herman MJ, Buck B, Pizzutillo PD. Hip fractures in children. J Am Acad Orthop Surg.
2009 Mar;17(3):162-73. Review. PubMed PMID: 19264709.
Beaty JH. Fractures of the hip in children. Orthop Clin North Am. 2006 Apr;37(2):223-32, vii. Review.
PubMed PMID: 16638453.

Figure 75

Question 75
Which muscles are responsible for the displacement of the proximal fragment of the fracture shown in
Figure 75?

1. Iliopsoas, hip abductors, hip external rotators


2. Iliopsoas, hip adductors, hip internal rotators
3. Rectus femoris, hip abductors, hip external rotators
4. Hamstrings, hip abductors, hip internal rotators

PREFERRED RESPONSE: 1

DISCUSSION
The radiograph shows a subtrochanteric femoral shaft fracture in a skeletally immature patient. The
proximal fragment is displaced into flexion, abduction, and external rotation. Flexion is attributable to the
pull of the iliopsoas at the lesser trochanter. Abduction is attributable to the pull of the abductor muscles
(gluteus medius and minimus) at the greater trochanter. External rotation is attributable to the pull of the
small external rotators, including the piriformis. The majority of the adductor musculature originates on
the symphysis pubis and bypasses the proximal femur, inserting further distally on the adductor tubercle.
The hamstrings originate on the ischial tuberosity and also bypass the proximal fragment, inserting distally
on the proximal tibia and fibula.

2013 American Academy of Orthopaedic Surgeons 2013 Pediatric Orthopaedic Self-Assessment Examination
74 American Academy of Orthopaedic Surgeons

RECOMMENDED READINGS
Femoral shaft fractures. In: Beaty JH, Kasser JR, eds. Rockwood and Wilkins Fractures in Children. 6th
ed. Philadelphia, PA: Lippincott Williams & Wilkins; 2006:chap 22.
Pring M, Newton P, Rang M. Femoral shaft. In: Rang M, Pring M, Wenger D, eds. Rangs Childrens
Fractures. Philadelphia, PA: Lippincott Williams & Wilkins; 2005:chap 13.

Question 76
What is the most common metatarsal fracture in a 3-year-old?

1. Proximal first metatarsal


2. Proximal fifth metatarsal
3. Distal second metatarsal
4. Distal fifth metatarsal

PREFERRED RESPONSE: 1

DISCUSSION
In children 5 years of age and younger, the proximal first metatarsal is the most common site for
metatarsal fractures; the fractures are usually caused by a fall. In children older than age 5, injury to the
fifth metatarsal is most common and is sustained during play or sports. Many fractures in children under
age 5 are not initially visible on radiograph. Knowing where to look for common injuries is mandatory
when treating young children and nonverbal patients.

RECOMMENDED READINGS
Singer G, Cichocki M, Schalamon J, Eberl R, Hllwarth ME. A study of metatarsal fractures in children. J
Bone Joint Surg Am. 2008 Apr;90(4):772-6. PubMed PMID: 18381315.
Owen RJ, Hickey FG, Finlay DB. A study of metatarsal fractures in children. Injury. 1995 Oct;26(8):537-
8. PubMed PMID: 8550143.

2013 American Academy of Orthopaedic Surgeons 2013 Pediatric OrthopaedicSelf-Assessment Examination


2013 Pediatric Orthopaedic Self-Assessment Examination Answer Book 75

Figure 77a Figure 77b Figure 77c

CLINICAL SITUATION FOR QUESTIONS 77 AND 78


Figures 77a through 77c are the preoperative and postoperative radiographs of a 13-year-old boy who had
sudden worsening of left hip pain following many months of mild pain. He was unable to walk because of
his pain and underwent screw fixation.

Question 77
Use of the shorter, anterior screw may result in

1. deformity of the acetabular labrum when the hip is flexed.


2. injury to the femoral artery.
3. chondrolysis.
4. lack of stabilization of the epiphysis.

PREFERRED RESPONSE: 1

2013 American Academy of Orthopaedic Surgeons 2013 Pediatric Orthopaedic Self-Assessment Examination
76 American Academy of Orthopaedic Surgeons

Figure 78a Figure 78b

Question 78
Six weeks later the boy remains uncomfortable and continues to use crutches for all ambulation. What do
the new radiographs seen in Figures 78a and 78b reveal?

1. Osteonecrosis
2. Chondrolysis
3. Fixation failure at the femoral neck
4. Screw cutout of the femoral head

PREFERRED RESPONSE: 3

DISCUSSION FOR QUESTIONS 77 AND 78


It has been demonstrated on a cadaver model that screw fixation of moderate and severe slipped capital
femoral epiphyses may result in screw impingement upon the acetabulum and labrum. This is likely when
the screw head on the anteroposterior view is seen to lie medial to the intertrochanteric line. Femoral
artery pseudoaneurysm has been reported when the screws are left long (projecting far from the bone) to
ease removal. Chondrolysis is associated with persistent penetration into the hip joint; both screws stop
well short of the articular surface. Many in vitro studies of slip models have demonstrated increased
strength of construct of two screws compared to one, although the clinical relevance can be questioned.
The radiographs show the screw heads firmly in the femoral head, with loss of fixation in the femoral
neck. Sanders and associates reported a series of 7 such failures and hypothesized that acute-on-chronic
slips may develop osteopenia of the femoral neck. All patients reported continued pain postoperatively
rather than the relief typically seen following surgical stabilization of the epiphysis. There is no
radiographic evidence of osteonecrosis or chondrolysis.

2013 American Academy of Orthopaedic Surgeons 2013 Pediatric OrthopaedicSelf-Assessment Examination


2013 Pediatric Orthopaedic Self-Assessment Examination Answer Book 77

RECOMMENDED READINGS FOR QUESTIONS 77 AND 78


Goodwin RC, Mahar AT, Oswald TS, Wenger DR. Screw head impingement after in situ fixation in
moderate and severe slipped capital femoral epiphysis. J Pediatr Orthop. 2007 Apr-May;27(3):319-25.
PubMed PMID: 17414018.
Herndon WA, Yngve DA, Janssen TP. Iatrogenic false aneurysm in slipped capital femoral epiphysis. J
Pediatr Orthop. 1984 Nov;4(6):754-5. PubMed PMID: 6511906.
Kishan S, Upasani V, Mahar A, Oka R, Odell T, Rohmiller M, Newton P, Wenger D. Biomechanical
stability of single-screw versus two-screw fixation of an unstable slipped capital femoral epiphysis model:
effect of screw position in the femoral neck. J Pediatr Orthop. 2006 Sep-Oct;26(5):601-5. PubMed PMID:
16932098.
Sanders JO, Smith WJ, Stanley EA, Bueche MJ, Karol LA, Chambers HG. Progressive slippage after
pinning for slipped capital femoral epiphysis. J Pediatr Orthop. 2002 Mar-Apr;22(2):239-43. PubMed
PMID: 11856939.
Loder RT, Aronsson DD, Weinstein SL, Breur GJ, Ganz R, Leunig M. Slipped capital femoral epiphysis.
Instr Course Lect. 2008;57:473-98. Review. PubMed PMID: 18399603.

END OF SERIES

Question 79
What is the most common physical examination finding in a patient with chronic painful spondylolysis?

1. Positive straight leg raise


2. Pain with forward flexion
3. Pain with lumbar extension
4. Absent tendo-Achilles reflex

PREFERRED RESPONSE: 3

DISCUSSION
Patients with spondylolysis typically demonstrate increased pain with lumbar extension, not with forward
flexion. In the absence of a disk herniation, a straight leg raise test result should be negative. Pain with
forward flexion is not common in spondylolysis, and without nerve root impingement there should be no
loss of the tendo-Achilles reflex.

RECOMMENDED READINGS
Luhmann SJ, Skaggs DL. Pediatric spine conditions. In: Lieberman JR, ed. AAOS Comprehensive
Orthopaedic Review. Rosemont, IL: American Academy of Orthopaedic Surgeons; 2009:245-265.
McCleary MD, Congeni JA. Current concepts in the diagnosis and treatment of spondylolysis in young
athletes. Curr Sports Med Rep. 2007 Jan;6(1):62-6. Review. PubMed PMID: 17212915.

2013 American Academy of Orthopaedic Surgeons 2013 Pediatric Orthopaedic Self-Assessment Examination
78 American Academy of Orthopaedic Surgeons

Question 80
Plain radiographs do not provide an accurate assessment of bone mineral density (BMD) until what
percentage of mineral has been lost?

1. 5%
2. 20%
3. 40%
4. 90%

PREFERRED RESPONSE: 3

DISCUSSION
Radiographic evidence of BMD loss is not apparent until 40% reduction. Osteopenia should not be ruled
out based on an apparently normal mineralized bone.

RECOMMENDED READINGS
Tortolani PJ, McCarthy EF, Sponseller PD. Bone mineral density deficiency in children. J Am Acad
Orthop Surg. 2002 Jan-Feb;10(1):57-66. Review. PubMed PMID: 11809051.
Binkovitz LA, Henwood MJ. Pediatric DXA: technique and interpretation. Pediatr Radiol. 2007
Jan;37(1):21-31. Epub 2006 May 20. Review. PubMed PMID: 16715219.

Figure 81a Figure 81b

Question 81
Figures 81a and 81b are the anteroposterior radiograph of the spine and neck and the 3-dimensional
reconstruction of the shoulders and spine of a 6-year-old girl. The childs parent reported that she cannot
keep up with her peers. She has difficulty with monkey bars and any activities that required her to raise
her arms above her shoulder. Her neck is short, broad, and tilted. What is the diagnosis?

1. Facial scapular humeral dystrophy


2. Turner syndrome
3. Sprengels deformity
4. Congenital muscular torticollis
PREFERRED RESPONSE: 3
2013 American Academy of Orthopaedic Surgeons 2013 Pediatric OrthopaedicSelf-Assessment Examination
2013 Pediatric Orthopaedic Self-Assessment Examination Answer Book 79

DISCUSSION
Sprengels deformity or congenital elevation of the scapula is the result of failure of caudal migration of
the scapula. Approximately 30% of patients with this condition have an omovertebral bone as seen in
the figures. In this patient the deformity is bilateral. Bilateral deformities occur in 10% to 30% of these
cases. Cartilage, fibrous tissue or bone (the omovertebral bone) may attach the scapula to the cervical
and thoracic spine. The latter limits shoulder abduction to 90 degrees. Cosmetically, the neck is broad
and webbed similar to the condition seen in Turner syndrome. Sprengels deformity is found in 35% of
patients with Klippel-Feil syndrome. Facial scapular humeral dystrophy does not entail an omoverterbral
bone as noted on this patients CT scan.

RECOMMENDED READINGS
Sarwark JF, LaBella CR, eds. Pediatric Orthopaedics and Sports Injuries: A Quick Reference Guide. Elk
Grove Village, IL: American Academy of Pediatrics; 2010:231-234.
Farsetti P, Weinstein SL, Caterini R, De Maio F, Ippolito E. Sprengels deformity: long-term follow-up
study of 22 cases. J Pediatr Orthop B. 2003 May;12(3):202-10. PubMed PMID: 12703036.

Figure 82a Figure 82b Figure 82c

Question 82
Figures 82a through 82c show the radiograph and 3-dimensional (3-D) CT scans of a 2-year-old boy
whose scoliosis has progressed 15 degrees during the past year. The child is clinically healthy. He has
been walking since 11 months of age. An MRI scan of the entire spine revealed no other anomalies. What
additional study is indicated?

1. Renal ultrasound
2. Blood cultures
3. Flexion-extension cervical spine radiographs
4. Platelet count

PREFERRED RESPONSE: 1

2013 American Academy of Orthopaedic Surgeons 2013 Pediatric Orthopaedic Self-Assessment Examination
80 American Academy of Orthopaedic Surgeons

DISCUSSION
Renal anomalies are found in as many as one-third of patients with congenital scoliosis, so a renal
ultrasound should be obtained. There may be other anomalies, including cardiac. There are no other
anomalies on MRI, so flexion-extension cervical spine radiographs are not indicated. There is no
associated marrow or platelet problem with hemivertebra. There is no indication for blood cultures
because this is a noninfection disorder. The radiographs and 3-D CT scans show a hemivertebra scoliosis
already beyond 45 degrees. Resection of the hemivertebra with stabilization is the indicated treatment.
The scoliosis will get worse with observation and bracing. Fusion posteriorly can only minimally correct
and not stop progression of the scoliosis.

RECOMMENDED READINGS
Ruf M, Jensen R, Letko L, Harms J. Hemivertebra resection and osteotomies in congenital spine
deformity. Spine (Phila Pa 1976). 2009 Aug 1;34(17):1791-9. PubMed PMID: 19644330.
Xu W, Yang S, Wu X, Claus C. Hemivertebra excision with short-segment spinal fusion through combined
anterior and posterior approaches for congenital spinal deformities in children. J Pediatr Orthop B. 2010
Nov;19(6):545-50. PubMed PMID: 20647938.

Question 83
At the time of arthroscopy, a 9-year-old boy was found to have a Watanabe type II discoid lateral
meniscus. What is the most appropriate treatment?

1. Saucerization of the mensicus only


2. Saucerization and stabilization of the mensicus
3. Stabilization of the meniscus only
4. Complete menisectomy

PREFERRED RESPONSE: 1

DISCUSSION
The Watanabe type II mensicus should only require saucerization for treatment because it is not unstable.
The Watanabe classification defines 3 types of discoid mensici. In type I (stable, complete), the block-
shaped lateral meniscus covers the entire lateral tibial plateau, whereas in type II (stable, partial), the
lateral meniscus covers 80% of the tibial plateau. Type III discoid menisci (unstable, ligament of
Wrisberg) appear to be normal except for a thickened posterior horn, but they lack posterior meniscal
attachments, including the meniscotibial (ie, coronary) ligament. The type III discoid meniscus is
stabilized only by the meniscofemoral ligament of Wrisberg. This results in hypermobility of the lateral
meniscus at the posterior horn, which pulls into the intercondylar notch with knee extension, resulting in
snapping knee syndrome. Complete menisectomy should be avoided if possible.

2013 American Academy of Orthopaedic Surgeons 2013 Pediatric OrthopaedicSelf-Assessment Examination


2013 Pediatric Orthopaedic Self-Assessment Examination Answer Book 81

RECOMMENDED READINGS
Kramer DE, Micheli LJ. Meniscal tears and discoid meniscus in children: diagnosis and treatment. J Am
Acad Orthop Surg. 2009 Nov;17(11):698-707. Review. PubMed PMID: 19880680.
Stanitski CL. Discoid meniscus. In: Micheli LJ, Kocher MS, eds. The Pediatric and Adolescent Knee.
Philadelphia, PA: Elsevier; 2006:260-272.
Watanabe M, Takeda S, Ikeuchi H. Atlas of Arthroscopy. 3rd ed. Tokyo, Japan: Springer-Verlag; 1979:75-
130.

Figure 84a Figure 84b

Question 84
Figures 84a and 84b are the anteroposterior and lateral radiographs of one of the feet of a 7-month-old
infant who had bilateral flat feet on examination that exhibited 40-degree dorsiflexion and 10-degree
plantar flexion at the ankles and no subtalar motion. What is the most appropriate next step in
management?

1. Urgent surgical correction


2. Obtain a neurologic workup that includes neural axis imaging
3. Obtain an MRI scan of the foot
4. Advise the parents that flat feet are normal in infants and observation is recommended

PREFERRED RESPONSE: 2

DISCUSSION
Vertical tali have a high association with neural axis abnormalities and genetic neurologic syndromes, so
a neurologic workup is the first step. The radiographs show rigid vertical tali that do not reduce in plantar
flexion. Although the feet may correct with serial casting and less-invasive surgery may be required,
this is less likely with rigid feet that do not reduce in plantar flexion. Flexible flat feet are common in
infants and toddlers, but vertical tali are not. Vertical tali can be differentiated on examination by the
lack of plantar flexion, subtalar motion, and a rocker-bottom appearance. An MRI scan will not provide
additional information.

2013 American Academy of Orthopaedic Surgeons 2013 Pediatric Orthopaedic Self-Assessment Examination
82 American Academy of Orthopaedic Surgeons

RECOMMENDED READINGS
Merrill LJ, Gurnett CA, Connolly AM, Pestronk A, Dobbs MB. Skeletal muscle abnormalities and genetic
factors related to vertical talus. Clin Orthop Relat Res. 2011 Apr;469(4):1167-74. Epub 2010 Jul 20.
PubMed PMID: 20645034.
Kasser JR. The foot. In: Morrissy RT, Weinstein SL, eds. Lovell and Winters Pediatric Orthopaedics. 6th
ed. Lippincott Williams & Wilkins; Philadelphia, PA: 2006:1257-1328.

Figure 85

Question 85
Figure 85 is the radiograph of a 3-year-old boy whose chief issue is knocked knees. His mother notes that
she has a similar condition and required multiple surgeries as a child. She states that her son walks with
an unsteady gait and is small for his age. He does not currently take any medications and is not under
medical care for any disorders. What is the most appropriate next treatment step?

1. Recommend bilateral valgus-producing proximal femoral osteotomies to correct coxa vara


2. Recommend bilateral medial distal femoral and proximal tibial hemiepiphyseal arrests to
correct genu valgum using guided growth
3. Obtain serum calcium, phosphorus, alkaline phosphatase, and vitamin D studies and refer the
patient to an endocrinologist for evaluation
4. Refer the patient to a geneticist to evaluate him for skeletal dysplasia

PREFERRED RESPONSE: 3

2013 American Academy of Orthopaedic Surgeons 2013 Pediatric OrthopaedicSelf-Assessment Examination


2013 Pediatric Orthopaedic Self-Assessment Examination Answer Book 83

DISCUSSION
The radiograph shows a patient with osteopenia; marked limb deformity, including bilateral coxa vara
and bilateral genu valgum; and extreme physeal widening, which is pathonomonic for rickets. Although
surgery to correct the limb deformities may be appropriate, a definitive diagnosis first needs to be
established, and, if possible, the patient needs to be treated medically. In cases in which limb realignment
surgery has been performed without proper medial correction of the metabolic bone disease, the
recurrence rate is high. Serum calcium, phosphorus, alkaline phosphatase, and vitamin D are appropriate
screening studies for diagnosis of metabolic bone disease, and treatment is most commonly performed by
an endocrinologist. A geneticist may play a role in establishing the cause of the disease, especially if there
is a hereditary component, but this step should not delay the consultation with endocrinology.

RECOMMENDED READINGS
Stevens PM, Klatt JB. Guided growth for pathological physes: radiographic improvement during
realignment. J Pediatr Orthop. 2008 Sep;28(6):632-9. PubMed PMID: 18724199.
Tortolani PJ, McCarthy EF, Sponseller PD. Bone mineral density deficiency in children. J Am Acad
Orthop Surg. 2002 Jan-Feb;10(1):57-66. Review. PubMed PMID: 11809051.

Question 86
Where is the physis with the highest growth rate (in mm per year) located?

1. Proximal humerus
2. Distal femur
3. Distal tibia
4. Distal radius

PREFERRED RESPONSE: 2

DISCUSSION
The distal femur averages 9 mm to 11 mm in growth per year, the proximal humerus 7 mm per year, the
distal tibia 4 mm to 5 mm per year, and the distal radius 5 mm to 6 mm per year.

RECOMMENDED READINGS
Rathjen KE, Birch JG. Physeal injuries and growth disturbances. In: Beaty JH, Kasser JR, eds. Rockwood
and Wilkins Fractures in Children. 6th ed. Philadelphia, PA: Lippincott Williams & Wilkins; 2006:99-
132.
Spencer SA, Widmann RF. Limb-length discrepancy and limb lengthening. In: Song KM, ed. Orthopaedic
Knowledge Update: Pediatrics 4. Rosemont, IL: American Academy of Orthopaedic Surgeons; 2011:219-
232.

2013 American Academy of Orthopaedic Surgeons 2013 Pediatric Orthopaedic Self-Assessment Examination
84 American Academy of Orthopaedic Surgeons

Figure 87

Question 87
Figure 87 is the pelvic radiograph of a newborn who had stiff bilateral talipes equinovarus and flexion
contractures of both wrists with ulnar deviation. The Ortolani test result is negative in both hips. The
newborn is to remain in the neonatal unit to address feeding issues for 1 to 2 weeks. What is the
recommended course of action?

1. A Pavlik harness
2. A rigid abduction brace
3. CT scan of the pelvis
4. No treatment or imaging of the hips for several months

PREFERRED RESPONSE: 4

DISCUSSION
This child has a syndrome of multiple deformities, most likely amyoplasia. The right hip shows a
teratologic dislocation with suggestion of pseudoacetabulum formation. It is appropriate to proceed with
treatment of the foot and hand deformities and plan for open reduction of the hip at 6 months of age or
older. Harness or brace treatment is unlikely to succeed in this high-riding teratologic dislocation. A CT
scan performed during the neonatal period would not be helpful.

RECOMMENDED READINGS
Bevan WP, Hall JG, Bamshad M, Staheli LT, Jaffe KM, Song K. Arthrogryposis multiplex congenita
(amyoplasia): an orthopaedic perspective. J Pediatr Orthop. 2007 Jul-Aug;27(5):594-600. Review.
PubMed PMID: 17585274.
Bernstein RM. Arthrogryposis and amyoplasia. J Am Acad Orthop Surg. 2002 Nov-Dec;10(6):417-24.
Review. PubMed PMID: 12470044.

2013 American Academy of Orthopaedic Surgeons 2013 Pediatric OrthopaedicSelf-Assessment Examination


2013 Pediatric Orthopaedic Self-Assessment Examination Answer Book 85

Figure 88

Question 88
Figure 88 is the anteroposterior elbow radiograph of an 8-month-old boy who had swelling and decreased
use of his left elbow. His neurovascular examination is intact and he has no fever or chills. What is the
most appropriate initial treatment?

1. Obtain an MRI scan


2. Apply a long-arm splint
3. Closed reduction with pinning
4. Open reduction with pinning

PREFERRED RESPONSE: 3

DISCUSSION
This child has a Salter-Harris-type I fracture of the distal humerus. Standard treatment is closed reduction
with or without internal fixation with pins. An MRI scan is typically not needed. Applying a splint
without reduction will leave the fracture to heal in a suboptimal position. Open reduction is rarely
necessary and would not be the most appropriate initial treatment.

RECOMMENDED READINGS
Shrader MW. Pediatric supracondylar fractures and pediatric physeal elbow fractures. Orthop Clin North
Am. 2008 Apr;39(2):163-71, v. Review. PubMed PMID: 18374807.
Skaggs DL. Elbow fractures in children: diagnosis and management. J Am Acad Orthop Surg. 1997
Nov;5(6):303-312. PubMed PMID: 10797226.

2013 American Academy of Orthopaedic Surgeons 2013 Pediatric Orthopaedic Self-Assessment Examination
86 American Academy of Orthopaedic Surgeons

Figure 89a Figure 89b Figure 89c

CLINICAL SITUATION FOR QUESTIONS 89 THROUGH 90


Figures 89a through 89c are the lateral radiograph and MRI scans of the right foot of a 15-year-old girl
who was born with terminal amputations of her right middle and ring fingers, a cleft lip and palate,
constriction bands at the left distal leg, and a right clubfoot for which she underwent surgical treatment at
age 2. She has a chronic draining ulcer of her distal fifth metatarsal.

Question 89
What is the most likely cause of her foot deformity and the ulcer?

1. Pin tract infection from fixation after posterior medial release


2. Inadequate abductor hallucis release
3. Inadequate lateral release
4. Right peroneal neuropathy

PREFERRED RESPONSE: 4

Question 90
What is the most likely diagnosis for her condition?

1. Congenital amniotic band syndrome


2. Ehlers-Danlos syndrome
3. Fibular hemimelia
4. Arthrogryposis multiplex congenital

PREFERRED RESPONSE: 1

2013 American Academy of Orthopaedic Surgeons 2013 Pediatric OrthopaedicSelf-Assessment Examination


2013 Pediatric Orthopaedic Self-Assessment Examination Answer Book 87

DISCUSSION FOR QUESTIONS 89 THROUGH 90


Streeter dysplasia or amniotic band syndrome is a congenital disorder caused by entrapment of fetal
parts (usually a limb or digits) in fibrous amniotic bands in utero. It is associated with cleft lip or
palate, terminal amputations, constriction bands, encephalocele, renal abnormalities, cardiac defects,
hemihypertrophy, anterolateral bowing, tibial pseudarthrosis, and limb-length discrepancy. Clubfoot is
seen in up to 25% of cases. Tight bands around the peroneal nerve that occur in 50% of children with
clubfeet can lead to nerve damage. This patient has chronic osteomyelitis resulting from pressure on the
fifth metacarpal head in the presence of an insensate lateral foot. Traditional pinning in open clubfoot
surgery does not breach the distal metatarsal. In an open procedure, releasing the abductor hallucis
addresses forefoot abduction and is performed medially. The lateral release impacts the midfoot and
hindfoot, but not lateral ray plantar flexion.

RECOMMENDED READINGS FOR QUESTIONS 89 THROUGH 90


Allington NJ, Kumar SJ, Guille JT. Clubfeet associated with congenital constriction bands of the
ipsilateral lower extremity. J Pediatr Orthop. 1995 Sep-Oct;15(5):599-603. PubMed PMID: 7593570.
Gomez VR. Clubfeet in congenital annular constricting bands. Clin Orthop Relat Res. 1996
Feb;(323):155-62. PubMed PMID: 8625573.
Do T. Streeter Dysplasia. http://emedicine.medscape.com/article/1260337-overview (Accessed 8/7/2012).

END OF SERIES

Question 91
A 13-year-old boy who has a history of a pituitary adenoma has an unstable unilateral slipped capital
femoral epiphysis. What is the indication for prophylactic pinning of the contralateral, unslipped side?

1. Patient gender
2. Patient age
3. Presentation with an unstable slipped epiphysis
4. Coexisting endocrine disorder

PREFERRED RESPONSE: 4

DISCUSSION
Endocrine disorders post the highest risk for bilateral involvement, and prophylactic pinning of the
uninvolved side is most often recommended. Risk of contralateral slippage is highest in the youngest
patients. In a study by Riad and associates, all girls younger than age 10 and all boys younger than age
12 presenting with a unilateral slipped capital femoral epiphysis subsequently developed a contralateral
slip. Initial presentation of an unstable slip has not been shown to be an independent risk factor for later
contralateral slippage.

2013 American Academy of Orthopaedic Surgeons 2013 Pediatric Orthopaedic Self-Assessment Examination
88 American Academy of Orthopaedic Surgeons

RECOMMENDED READINGS
Wells D, King JD, Roe TF, Kaufman FR. Review of slipped capital femoral epiphysis associated with
endocrine disease. J Pediatr Orthop. 1993 Sep-Oct;13(5):610-4. Review. PubMed PMID: 8376562.
Riad J, Bajelidze G, Gabos PG. Bilateral slipped capital femoral epiphysis: predictive factors for
contralateral slip. J Pediatr Orthop. 2007 Jun;27(4):411-4. PubMed PMID: 17513962.
Podeszwa DA, Sucato DJ. Slipped capital femoral epiphysis. In: Song KM, ed. Orthopaedic Knowledge
Update: Pediatrics 4. Rosemont, IL: American Academy of Orthopaedic Surgeons; 2011:187-194.

Figure 92

Question 92
Figure 92 is the CT scan of an 8-month-old girl with a left developmental hip dislocation who underwent
a closed reduction with adductor longus tenotomy. An intraoperative arthrogram confirmed an adequate
reduction and the child was placed in a hip spica cast. What is the most appropriate treatment?

1. Return to the operating room in 6 weeks for a cast change under anesthesia
2. Urgent repeat closed reduction and casting
3. Open reduction and femoral shortening osteotomy
4. Open reduction and Salter innominate osteotomy

PREFERRED RESPONSE: 2

DISCUSSION
The CT scan shows the left femoral head is subluxated posteriorly. Because reduction was adequate in
the operating room, the best approach is to return to the operating room expeditiously, repeat the closed
reduction, and recast in a better position. Open reduction could be considered if closed treatment is
unsuccessful, but femoral shortening or innominate osteotomy should not be required at this age.

RECOMMENDED READINGS
MacDonald J, Barrow S, Carty HM, Taylor JF. Imaging strategies in the first 12 months after reduction of
developmental dislocation of the hip. J Pediatr Orthop B. 1995;4(1):95-9. PubMed PMID: 7719843.
Chin MS, Betz BW, Halanski MA. Comparison of hip reduction using magnetic resonance imaging or
computed tomography in hip dysplasia. J Pediatr Orthop. 2011 Jul-Aug;31(5):525-9. PubMed PMID:
21654460.

2013 American Academy of Orthopaedic Surgeons 2013 Pediatric OrthopaedicSelf-Assessment Examination


2013 Pediatric Orthopaedic Self-Assessment Examination Answer Book 89

Figure 93

Question 93
Figure 93 is the radiograph of 3-year-old girl who was evaluated for a progressive increase in tibia vara
and complains that her feet are turning in. What is the most appropriate course of action?

1. Bilateral tibia and fibular osteotomies


2. Valgus positioning knee-ankle-foot orthosis (KAFO)
3. Use of a Denis-Browne bar
4. Schedule a return visit in 4 months

PREFERRED RESPONSE: 1

DISCUSSION
The radiograph shows advanced changes in the medial tibial growth plates. The metaphyseal-diaphyseal
angle is more than 20 degrees. With these advanced changes, an osteotomy is indicated. Waiting will
allow the problem to increase. Bracing (KAFO or night bar) has not been shown effective in advanced
Blount disease.

RECOMMENDED READINGS
Sabharwal S. Blount disease. J Bone Joint Surg Am. 2009 Jul;91(7):1758-76. Review. PubMed PMID:
19571101.
Loder RT, Johnston CE II. Infantile tibia vara. J Pediatr Orthop. 1987 Nov-Dec;7(6):639-46. PubMed
PMID: 3429646.

2013 American Academy of Orthopaedic Surgeons 2013 Pediatric Orthopaedic Self-Assessment Examination
90 American Academy of Orthopaedic Surgeons

Figure 94

Question 94
An 18-month-old girl sustained the isolated injury seen in Figure 94. She had pain and deformity at the
fracture site and intact motor function and sensation distal to the injury. What is the most appropriate
treatment?

1. Open reduction and internal fixation


2. Closed reduction and percutaneous pinning
3. Splinting and use of a sling for comfort
4. Application of a hanging-arm cast

PREFERRED RESPONSE: 3

DISCUSSION
The radiograph shows a left midshaft humerus fracture in a young, skeletally immature patient. The
fracture does show significant angulation at age 18 months; however, even if the fracture heals with a
malunion, excellent remodeling can be expected. As a result, none of the invasive treatment methods are
necessary. Application of a hanging-arm cast likely would not provide any advantage in such a young
patient who probably does not spend much time in a sitting or standing position. Any humeral shaft
fracture in a nonambulatory patient should prompt consideration of nonaccidental trauma.

RECOMMENDED READINGS
Pring M, Wenger D. Shoulder and humeral shaft. In: Rang M, Pring M, Wenger D, eds. Rangs Childrens
Fractures. Philadelphia, PA: Lippincott Williams & Wilkins; 2005:chap 7.
Beaty JH. Fractures of the proximal humerus and shaft in children. Instr Course Lect. 1992;41:369-72.
PubMed PMID: 1588080.

2013 American Academy of Orthopaedic Surgeons 2013 Pediatric OrthopaedicSelf-Assessment Examination


2013 Pediatric Orthopaedic Self-Assessment Examination Answer Book 91

Question 95
An 11-year-old boy with bipolar disorder fell from a tree and sustained an open fracture dislocation of
the right ankle with extensive abrasions of the leg. Immediate irrigation, debridement, reduction, and
provisional fixation with Kirschner wires was performed. Twenty-four hours later, the patients blood
pressure is 190/100 mm Hg and pulse rate is 120. He has required only 1 dose of an oral analgesic for
pain control. His foot and ankle are markedly swollen, but there is no pain on passive extension of the
toes. The dorsalis pedis pulse cannot be palpated. What is the most appropriate next treatment step?

1. Remove the Kirschner wires and reposition the ankle


2. Measure foot compartment pressures
3. Order an echocardiogram
4. Order serum blood urea nitrogen and creatinine levels

PREFERRED RESPONSE: 2

DISCUSSION
The most common symptom of compartment syndrome in the extremities is intense pain. Compartment
syndrome can be difficult to diagnose in children and patients who are comatose, nonverbal, and/
or mentally compromised because they may not be able to properly express their level of pain. In
compartment syndrome of the leg, pain on passive extension of the toes is the most frequent clinical
diagnostic finding. However, in compartment syndrome of the foot, pain on passive extension of the toes
may or may not be present. Swelling and absence of the dorsalis pedis pulse may be expected findings
with extensive trauma to the foot, making the clinical diagnosis even more difficult. Repositioning the
ankle will add to further swelling. The clinician must be alerted regarding elevations in blood pressure
and pulse because such elevations may be the only manifestation of the deeper problem. The transient
blood pressure elevation does not require cardiac screening with electrocardiogram or echocardiogram as
in chronic hypertension. Kidney function testing is not necessary because the blood pressure elevation is
not renal in origin. Compartment pressures should be measured immediately in the foot and will require
anesthesia in the pediatric age group.

RECOMMENDED READINGS
Ojike NI, Roberts CS, Giannoudis PV. Foot compartment syndrome: a systematic review of the literature.
Acta Orthop Belg. 2009 Oct;75(5):573-80. Review. PubMed PMID: 19999866.
Badhe S, Baiju D, Elliot R, Rowles J, Calthorpe D. The silent compartment syndrome. Injury. 2009
Feb;40(2):220-2. Epub 2009 Feb 8. PubMed PMID: 19203751.
Flynn JM, Bashyal RK, Yeger-McKeever M, Garner MR, Launay F, Sponseller PD. Acute traumatic
compartment syndrome of the leg in children: diagnosis and outcome. J Bone Joint Surg Am. 2011 May
18;93(10):937-41. PubMed PMID: 21593369.

2013 American Academy of Orthopaedic Surgeons 2013 Pediatric Orthopaedic Self-Assessment Examination
92 American Academy of Orthopaedic Surgeons

Question 96
What is a known risk factor for wound infection after spinal fusion for neuromuscular scoliosis?

1. Number of levels fused


2. Blood loss
3. Serum albumin level < 3.5 g/dL
4. Preoperative curve magnitude

PREFERRED RESPONSE: 3

DISCUSSION
The risk for wound infection after spinal fusion for neuromuscular scoliosis ranges from 4% to 14% and
is higher than risk after spinal fusion in idiopathic scoliosis. A recent study of a database of 151 patients
with neuromuscular scoliosis found the presence of ventriculoperitoneal shunt to be associated with an
increased risk for wound infection. Age, preoperative major curve magnitude, number of vertebral levels
fused, length of surgery, blood loss, and transfusion requirements were not associated with increased risk.
A previous study found that poor nutritional status as measured by serum albumin < 3.5 g/dL (reference
range, 3.5-5.0 g/dL) or lymphocytes < 1500 cells/L (reference range, 1000-4800/L) has been associated
with increased postoperative wound infections.

RECOMMENDED READINGS
Master DL, Connie PK, Jochen SH, Armstrong DG, Thompson GH. Wound infections after surgery
for neuromuscular scoliosis: risk factors and treatment outcomes. Spine (Phila Pa 1976). 2011 Feb
1;36(3):E179-85. PubMed PMID: 21248588.
Szke G, Lipton G, Miller F, Dabney K. Wound infection after spinal fusion in children with cerebral
palsy. J Pediatr Orthop. 1998 Nov-Dec;18(6):727-33. PubMed PMID: 9821126.
Jevsevar DS, Karlin LI. The relationship between preoperative nutritional status and complications after
an operation for scoliosis in patients who have cerebral palsy. J Bone Joint Surg Am. 1993 Jun;75(6):880-
4. Erratum in: J Bone Joint Surg Am 1993 Aug;75(8):1256. PubMed PMID: 8314827.

Question 97
A 2-year-old boy was evaluated for toe-walking and clumsiness that were not present when he first walked
at 1 year. His calves were noted to be quite large. He had a rigid equinus contracture of 10 degrees.
When getting up from the floor, he rose to a tripod position and walked his hands up his legs until he was
upright. Which of the following tests is most useful at this time?

1. Radiographs of his legs


2. Nerve conduction testing of his peripheral nerves
3. Creatine kinase blood level
4. MRI scan of his brain and spine

PREFERRED RESPONSE: 3

2013 American Academy of Orthopaedic Surgeons 2013 Pediatric OrthopaedicSelf-Assessment Examination


2013 Pediatric Orthopaedic Self-Assessment Examination Answer Book 93

DISCUSSION
This history is classic for Duchenne muscular dystrophy, a condition present in approximately 1 in 3500
live male births. The disorder is caused by absent dystrophin in muscle; consequently, the creatine kinase
levels at symptom onset are quite high (often > 25000 IU/L) (reference range, 55-365 IU/L). This is not
a nerve disorder, so nerve tests, radiographs of the legs, and MRI scans of the neural axis will not yield a
diagnosis.

RECOMMENDED READINGS
Thompson GH, Berenson FR. Other neuromuscular disorders. In: Morrissy RT, Weinstein SL, eds. Lovell
and Winters Pediatric Orthopaedics. 6th ed. Philadelphia, PA: Lippincott Williams & Wilkins; 2006:649-
692.
Sussman M. Duchenne muscular dystrophy. J Am Acad Orthop Surg. 2002 Mar-Apr;10(2):138-51.
Review. PubMed PMID: 11929208.

Question 98
A dorsal bunion is a complication of clubfoot treatment. The pathophysiology is believed to be a muscle
imbalance between

1. a weak anterior tibial muscle and strong peroneus longus muscle.


2. weak triceps and strong toe flexors.
3. strong triceps and weak toe flexors.
4. strong anterior tibial and strong toe flexors.

PREFERRED RESPONSE: 2

DISCUSSION
Two mechanisms can create a dorsal bunion in a treated clubfoot. In the clubfoot, strong toe flexors,
flexor hallucis brevis, and abductor hallucis try to compensate for a weak triceps during push off. The
classic mechanism is a strong anterior tibial dorsiflexing the first metatarsal, which is unopposed by weak
peroneals. The other responses would not produce this deformity.

RECOMMENDED READINGS
Herring JA, ed. Tachdjians Pediatric Orthopaedics. 4th ed. Philadelphia, PA: Saunders Elsevier;
2008:376-379.
Beaty JH. Congenital anomolies of the lower extremity. In: Canale ST, Beaty JH, eds. Campbells
Operative Orthopaedics. 11th ed. Philadelphia, PA: Mosby; 2008:263-270.

2013 American Academy of Orthopaedic Surgeons 2013 Pediatric Orthopaedic Self-Assessment Examination
94 American Academy of Orthopaedic Surgeons

Question 99
A 3-year-old child with achondroplasia is hypotonic, and sleep studies reveal central apneic episodes. An
initial workup should evaluate for what diagnosis?

1. Foramen magnum stenosis


2. C1-C2 instability
3. Adenoid and tonsillar hypertrophy
4. Tethered spinal cord

PREFERRED RESPONSE: 1

DISCUSSION
Foramen magnum stenosis can cause sudden death in young children with achondroplasia and must
be ruled out with CT or MRI scans. C1-C2 instability does not occur in patients with achondroplasia.
Lumbar stenosis and thoracolumbar kyphosis can be seen but are not life threatening and would not
cause apnea or global hypotonia. Although many children with achondroplasia have adenoid and
tonsillar hypertrophy, this condition causes obstructive rather than central sleep apnea and does not cause
hypotonia. A tethered spinal cord is not particularly associated with achondroplasia and would not cause
central apnea.

RECOMMENDED READINGS
Shirley ED, Ain MC. Achondroplasia: manifestations and treatment. J Am Acad Orthop Surg. 2009
Apr;17(4):231-41. Review. PubMed PMID: 19307672.
Sponseller PD, Ain MC. The skeletal dysplasias. In: Morrissy RT, Weinstein SL, eds. Lovell and Winters
Pediatric Orthopaedics. 6th ed. Philadelphia, PA: Lippincott Williams & Wilkins; 2006:205-250.

RESPONSES FOR QUESTIONS 100 THROUGH 104


1- Borrelia titer
2- Calcium, phosporus, alkaline phosphatase, vitamin D
3- Human leukocyte antigen (HLA)-B27
4- Complete blood count, erythrocyte sedimentation rate, C-reactive protein
5- Rheumatoid factor
6- Antinuclear antibody
7- Uric acid

The laboratory studies listed above are needed to diagnose the disorders listed below. Match the
appropriate laboratory studies with each of these disorders.

2013 American Academy of Orthopaedic Surgeons 2013 Pediatric OrthopaedicSelf-Assessment Examination


2013 Pediatric Orthopaedic Self-Assessment Examination Answer Book 95

Question 100
A 4-year-old girl has a 3-day history of progressive knee pain and the inability to bear weight. Her mother
notes that she had a minor fall last week, but at that time her ambulation was not inhibited. Her physical
examination reveals no knee effusion and tenderness in the distal femoral metaphysis. She has a low-
grade fever. Knee radiograph findings are normal.

PREFERRED RESPONSE: 4

Question 101
A 5-year-old boy has a 3-month history of pain in his left knee. His mother notes that he wakes each
morning complaining of pain and stiffness. He has trouble walking to the breakfast table. His mother
reports that the pain improves with ibuprofen, that he is able to go to school, and that he does not complain
in the afternoons or evenings. His physical examination reveals mild swelling of the left knee compared
to the right, but no effusion. Range of motion of the knee is painless. He does not have any bony
tenderness. Knee radiograph findings are normal.

PREFERRED RESPONSE: 4

Question 102
A 17-year-old boy has a 6-month history of low-back pain and stiffness. He localizes the pain to the lower
lumbar paraspinous muscles. He has tried ibuprofen, experiencing some relief. His physical examination
reveals pain in the low back with flexion, abduction, and external rotation of the hips. His radiographs
reveal increased sclerosis adjacent to the sacroiliac joints.

PREFERRED RESPONSE: 3

Question 103
A 14-month-old African-American boy is evaluated for bowed legs. His mother notes no family history
of limb deformities. He has been walking for 3 months. His mother notes that the bowing seems to be
worsening. She reports that he has gained weight and met his motor milestones appropriately. He eats
well but remains exclusively breastfed. His radiographs reveal osteopenia and widening of the physes at
the distal femurs and proximal tibias.

PREFERRED RESPONSE: 2

2013 American Academy of Orthopaedic Surgeons 2013 Pediatric Orthopaedic Self-Assessment Examination
96 American Academy of Orthopaedic Surgeons

Question 104
A 9-year-old boy returned from summer camp 4 months ago. He now has a 4-week history of right knee
swelling. He denies significant pain, history of trauma, or any recent fevers or illness. His physical
examination reveals a large knee joint effusion, painless range of motion of the knee, and stable knee
ligaments.

PREFERRED RESPONSE: 1

DISCUSSION FOR QUESTIONS 100 THROUGH 104


The patient in Question 100 has a history most consistent with an infectious process such as osteomyelitis.
Osteomyelitis most often presents acutely in children and is frequently associated with a precipitating
minor trauma. For the patient in Question 101 these findings are more consistent with juvenile idiopathic
arthritis. Complete blood count, erythrocyte sedimentation rate, and C-reactive protein labs are indicated
as initial testing in inflammatory conditions, including infection and juvenile idiopathic arthritis. Unlike
adults who have rheumatoid arthritis, children with juvenile idiopathic arthritis are not likely to have a
positive rheumatoid factor and testing does not need to be routinely ordered. The patient in Question
102 has a history and examination suggestive of ankylosing spondylitis. HLA-B27 has 92% sensitivity
and specificity for ankylosing spondylitis in European Americans and is present in 90% of affected
individuals. Alkaline phosphatase, along with serum calcium, phosphorus, and vitamin D, are essential
to screen for metabolic bone disorders, including rickets. The presentation of the child in Question 103
is most consistent with nutritional rickets. Although rare in the United States because multiple dietary
supplements are easily accessible, nutritional rickets still occurs, most commonly in exclusively breastfed
children (human breast milk does not provide significant vitamin D) who are dark skinned (vitamin D is
synthesized in the skin from ultraviolet light exposure, and dark-skinned children absorb less UV light
than light-skinned children). The patient in Question 104 has a history and examination suggestive of
Lyme arthritis. Lyme disease is caused by the bacterium borrelia burgdorferi, which is transferred to
humans through deer tick bites, most often in the Northeastern United States. Lyme arthritis usually
presents as a relatively painless knee effusion. A titer for this organism, which is initially an enzyme-
linked immunosorbent assay, is appropriate. Antinuclear antibodies are most commonly ordered when
there is suspicion for systemic lupus erythematosis. Uric acid is most commonly ordered when there is
suspicion for gouty arthritis.

RECOMMENDED READINGS FOR QUESTIONS 100 THROUGH 104


Punaro M. Rheumatologic conditions in children who may present to the orthopaedic surgeon. J Am Acad
Orthop Surg. 2011 Mar;19(3):163-9. Review. PubMed PMID: 21368097.
Copley LA. Pediatric musculoskeletal infection: trends and antibiotic recommendations. J Am Acad
Orthop Surg. 2009 Oct;17(10):618-26. Review. PubMed PMID: 19794219.
Gardner GC, Kadel NJ. Ordering and interpreting rheumatologic laboratory tests. J Am Acad Orthop Surg.
2003 Jan-Feb;11(1):60-7. Review. PubMed PMID: 12699372.
Tortolani PJ, McCarthy EF, Sponseller PD. Bone mineral density deficiency in children. J Am Acad
Orthop Surg. 2002 Jan-Feb;10(1):57-66. Review. PubMed PMID: 11809051.
Smith BG, Cruz AI Jr, Milewski MD, Shapiro ED. Lyme disease and the orthopaedic implications of lyme
arthritis. J Am Acad Orthop Surg. 2011 Feb;19(2):91-100. Review. PubMed PMID: 21292932.

END OF SERIES
2013 American Academy of Orthopaedic Surgeons 2013 Pediatric OrthopaedicSelf-Assessment Examination
2013 Pediatric Orthopaedic Self-Assessment Examination Answer Book 97

Figure 105a Figure 105b Figure 105c

Question 105
Figures 105a through 105c are the radiographs of a 13-year-old girl with right medial foot pain. The pain
has been present for 3 months, is exacerbated by walking and running, and is relieved by rest. She had
tried over-the-counter arch supports and nonsteroidal anti-inflammatory drugs for pain relief, but without
success. Surgical treatment has been recommended. Which surgical procedure is most likely to relieve
the patients symptoms?

1. Advancement of the posterior tibialis tendon to the inferior portion of the navicular
2. Splitting of the posterior tibialis tendon to excise the bony prominence
3. Performing a hindfoot osteotomy to correct pes planus
4. Performing osteosynthesis of the nonunited bone

PREFERRED RESPONSE: 2

DISCUSSION
Studies that have compared simple excision of the painful accessory navicular and prominent medial
navicular with advancement of the posterior tibialis into the plantar midfoot (Kidner procedure) have
shown no significant difference in outcomes. Simple excision, without correction of hindfoot valgus,
has been shown to provide adequate pain relief in the majority of patients. Fixation of the nonunited
accessory navicular to the proper navicular will not decrease the size of the painful medial prominence.

RECOMMENDED READINGS
Macnicol MF, Voutsinas S. Surgical treatment of the symptomatic accessory navicular. J Bone Joint Surg
Br. 1984 Mar;66(2):218-26. PubMed PMID: 6707058.
Kopp FJ, Marcus RE. Clinical outcome of surgical treatment of the symptomatic accessory navicular. Foot
Ankle Int. 2004 Jan;25(1):27-30. PubMed PMID: 14768961.

2013 American Academy of Orthopaedic Surgeons 2013 Pediatric Orthopaedic Self-Assessment Examination
98 American Academy of Orthopaedic Surgeons

Question 106
Which gene correlates with severity of disease in spinal muscular atrophy (SMA)?

1. Peripheral myelin protein 22 (PMP22)


2. Survival motor neuron I (SMN1)
3. Survival motor neuron II (SMN2)
4. Dystrophin

PREFERRED RESPONSE: 3

DISCUSSION
SMA is caused by a deficiency in SMN protein. Deficiency of SMN protein leads to progressive loss of
anterior horn cells and progressive muscle weakness. The severity of disease is directly related to the
amount of reduction in circulating levels of SMN proteins, which are encoded by 2 alleles of the SMN1
gene and multiple copies of the SMN2 genes on chromosome 5. Affected patients with all types of SMA
will have functional loss of both SMN1 genes, so this does not differentiate disease severity. Disease
severity depends on the number of functional copies of SMN2 that remain. Patients with SMA1 have
only 1 functioning SMN2 gene, whereas the milder forms, SMA types 2 and 3, have multiple copies that
produce higher levels of SMN protein. The other choices are not associated with spinal muscular atrophy.
Defects in PMP22 are the cause of 70% to 80% of cases of Charcot-Marie-Tooth disease. Mutations in
the dystrophin gene cause Duchenne muscular dystrophy.

RECOMMENDED READINGS
Sucato DJ. Spine deformity in spinal muscular atrophy. J Bone Joint Surg Am. 2007 Feb;89 Suppl 1:148-
54. Review. Erratum in: J Bone Joint Surg Am. 2007 May;89(5):1090-1. PubMed PMID: 17272431.
Sussman MD. Progressive neuromuscular diseases. In: Abel MF, ed. Orthopaedic Knowledge Update:
Pediatrics 3. Rosemont, IL: American Academy of Orthopaedic Surgeons; 2006:123-135.

Question 107
A 6-year-old girl is brought in for orthopaedic examination because she walks funny and occasionally
trips. Her feet are normal in appearance. Internal rotation of her hips is 60 degrees and external rotation is
40 degrees bilaterally. The thigh-foot angle on the right is -20 degrees, and +10 degrees on the left. What
is the source of her gait abnormality?

1. Increased femoral anteversion


2. Increased femoral retroversion
3. Internal tibial torsion, right
4. External tibial torsion, left

PREFERRED RESPONSE: 3

2013 American Academy of Orthopaedic Surgeons 2013 Pediatric OrthopaedicSelf-Assessment Examination


2013 Pediatric Orthopaedic Self-Assessment Examination Answer Book 99

DISCUSSION
Normal rotation in the hips should total approximately 100 degrees. Hip internal rotation is usually
greater than external rotation, especially in girls. The thigh-foot angle is measured in a prone position and
measures the angle the midaxis of the foot subtends with the midline of the thigh. The thigh-foot angle
quantifies tibial rotation. Normal adult thigh-foot angle is approximately 10 to 15 degrees. This patient
has increased internal rotation of the right tibia compared with the left; this is the likely cause of her gait
problem.

RECOMMENDED READINGS
Kling TF Jr, Hensinger RN. Angular and torsional deformities of the lower limbs in children. Clin Orthop
Relat Res. 1983 Jun;(176):136-47. PubMed PMID: 6851317.
Staheli LT, Engel GM. Tibial torsion: a method of assessment and a survey of normal children. Clin
Orthop Relat Res. 1972 Jul-Aug;86:183-6. PubMed PMID: 5047787.

Question 108
A 15-year-old boy with mild type I osteogenesis imperfecta (OI) has a midshaft radius/ulna fracture that is
in bayonet apposition with loss of the radial bow and 40-degree apex volar and ulnar angulation. Closed
reduction improves the angulation to 20 degrees; the bayonet apposition and loss of radial bow remains.
His contralateral forearm has a normal appearance upon examination. What is the best treatment for this
fracture?

1. Open reduction and plate fixation


2. Open reduction and intramedullary rod fixation with casting
3. Cast immobilization with expected remodeling of the fracture and near-full motion
4. Cast immobilization, accepting malunion and some dysfunction because surgical treatment has
a high rate of nonunion in OI

PREFERRED RESPONSE: 2

DISCUSSION
Teenagers with displaced midshaft forearm fractures do not remodel fully if the angulation is beyond 10
degrees after reduction. Excellent outcomes have been shown with intramedullary rodding as well as
plate fixation. In patients with bone fragility, plate fixation poses higher risks for fracture attributable
to stress shielding. Load sharing with intramedullary rodding is preferred in surgical management of
fractures whenever possible. This patient is 15 years old with unacceptable reduction, so remodeling of
this fracture would not be expected with closed management. Patients with OI are not at higher risk for
nonunion with surgery; there is no reason to withhold surgery for this patient.

RECOMMENDED READINGS
Zeitlin L, Fassier F, Glorieux FH. Modern approach to children with osteogenesis imperfecta. J Pediatr
Orthop B. 2003 Mar;12(2):77-87. Review. PubMed PMID: 12584489.
Noonan KJ, Price CT. Forearm and distal radius fractures in children. J Am Acad Orthop Surg. 1998 May-
Jun;6(3):146-56. Review. PubMed PMID: 9689186.

2013 American Academy of Orthopaedic Surgeons 2013 Pediatric Orthopaedic Self-Assessment Examination
100 American Academy of Orthopaedic Surgeons

Figure 109

Question 109
Figure 109 is the radiograph of an 11-year-old boy who felt a snap in his right hip while jumping hurdles
during track practice yesterday. He complains of pain to his right groin region and is walking with a limp.
What physical examination test will cause the patient to experience the most discomfort?

1. Resisted hip adduction


2. Resisted hip abduction
3. Resisted hip extension
4. Resisted knee extension

PREFERRED RESPONSE: 4

DISCUSSION
The radiograph shows an avulsion fracture from the right anterior inferior iliac spine. This is the site
of origin of the rectus femoris tendon. Contraction of the rectus femoris is most pronounced with
extension of the knee. The adductor muscles, which would be tested with resisted hip adduction, originate
predominantly on the symphysis pubis. The abductors, which would be tested with resisted hip abduction,
originate on the outer iliac crest. The hip extensors, which would be tested with resisted hip extension,
originate on the posterior iliac crest.

RECOMMENDED READINGS
Rossi F, Dragoni S. Acute avulsion fractures of the pelvis in adolescent competitive athletes: prevalence,
location and sports distribution of 203 cases collected. Skeletal Radiol. 2001 Mar;30(3):127-31. PubMed
PMID: 11357449.
Reina N, Accadbled F, de Gauzy JS. Anterior inferior iliac spine avulsion fracture: a case report in soccer
playing adolescent twins. J Pediatr Orthop B. 2010 Mar;19(2):158-60. PubMed PMID: 19934773.

2013 American Academy of Orthopaedic Surgeons 2013 Pediatric OrthopaedicSelf-Assessment Examination


2013 Pediatric Orthopaedic Self-Assessment Examination Answer Book 101

Figure 110

Question 110
Figure 110 is the hips-to-ankles radiograph of a 3-year-old girl being evaluated for bow legs. Her
examination reveals her body mass index is 35, and she has a left knee varus alignment with a lateral
thrust in gait. What is the most appropriate treatment?

1. Follow-up appointment in 6 months


2. Endocrine workup for rickets with calcium and vitamin D levels
3. Knee-ankle-foot orthosis brace to be worn during sports and follow-up in 6 months
4. Lateral hemiepiphysiodesis with guided growth technique

PREFERRED RESPONSE: 4

DISCUSSION
This patient has left knee early-stage infantile Blount disease. After age 2, physiologic genu varum
should be resolving. By age 3 affected children have physiologic genu valgum, so observation is not
appropriate because her medial physeal disturbance will progress to later-stage Blount disease with a bony
bar. Infantile Blount disease is believed to be a mechanical disorder and does not need endocrine workup;
also, the radiographs show no signs of rickets (physeal widening/cupping). Braces can be used, but they
are associated with success rates of 50% to 70% if worn appropriately and are most effective for children
younger than age 3. To be effective, bracing is best worn full time, and has not been shown effective
when worn fewer than 12 hours per day. Because recurrence rates after surgery are much higher in later-
stage Blount disease and in children older than age 4, a hemiepiphysiodesis should be done now. Lateral
hemiepiphysiodesis in Blount disease can be successful if done during the early stage and is minimally
invasive. Corrective osteotomy also would be a reasonable choice for treatment.

RECOMMENDED READINGS
Castaeda P, Urquhart B, Sullivan E, Haynes RJ. Hemiepiphysiodesis for the correction of angular
deformity about the knee. J Pediatr Orthop. 2008 Mar;28(2):188-91. PubMed PMID: 18388714.
Schoenecker PL, Rich MM. The lower extremity. In: Morrissy RT, Weinstein SL, eds. Lovell and Winters
Pediatric Orthopaedics. 6th ed. Philadelphia, PA: Lippincott Williams & Wilkins; 2006:1157-1212.

2013 American Academy of Orthopaedic Surgeons 2013 Pediatric Orthopaedic Self-Assessment Examination
102 American Academy of Orthopaedic Surgeons

Question 111
The Coleman block test is used to test for

1. flexibility of the forefoot.


2. flexibility of the hindfoot.
3. valgus deformity of the hindfoot.
4. tightness of tendo Achilles.

PREFERRED RESPONSE: 2

DISCUSSION
The Coleman block test is used to determine the flexibility of the hindfoot. When a block is placed under
the lateral border of the foot, the medial column is unsupported. As a result, the first metatarsal drops off
the side of the block. If the subtalar joint is flexible, there is no fixed varus deformity of the hindfoot. The
hindfoot will no longer be in varus from behind. The varus deformity of the hindfoot will be corrected. If
there is no subtalar motion, the varus deformity remains fixed.

RECOMMENDED READINGS
Paulos L, Coleman SS, Samuelson KM. Pes cavovarus. Review of a surgical approach using selective
soft-tissue procedures. J Bone Joint Surg Am. 1980 Sep;62(6):942-53. PubMed PMID: 7430182.
Pfeffer JB. Charcot-Marie-Tooth Disease. In: Pfeffer JB, Easly MA, Fray C, Hindenman B, Sans KN, eds.
Techniques in Foot and Ankle Surgery. Philadelphia, PA: Saunders; 2010:200.

Figure 112

Question 112
Figure 112 is the photograph of a 2-month-old infant with a left leg deformity. The mothers pregnancy
and delivery were unremarkable, and the infant is otherwise healthy. What is the most appropriate course
of action?

1. Osteotomy of the tibia and fibula


2. Clamshell knee-ankle-foot orthotic
3. Serial casts
4. Observation and a repeat visit in 4 months

PREFERRED RESPONSE: 4

2013 American Academy of Orthopaedic Surgeons 2013 Pediatric OrthopaedicSelf-Assessment Examination


2013 Pediatric Orthopaedic Self-Assessment Examination Answer Book 103

DISCUSSION
The photograph shows posteromedial bowing of the tibia. The child should be followed for later limb-
length discrepancy, which may need treatment. Treatment with bracing or serial casts has not been shown
to alter this condition. Surgery is not indicated in infancy because much of the deformity corrects with
time. At a later age, treatment of a limb-length discrepancy or residual deformity can be addressed.

RECOMMENDED READINGS
Shah HH, Doddabasappa SN, Joseph B. Congenital posteromedial bowing of the tibia: a retrospective
analysis of growth abnormalities in the leg. J Pediatr Orthop B. 2009 May;18(3):120-8. PubMed PMID:
19339901.
Pappas AM. Congenital posteromedial bowing of the tibia and fibula. J Pediatr Orthop. 1984
Sep;4(5):525-31. PubMed PMID: 6490868.

Figure 113a Figure 113b

Question 113
Figures 113a and 113b are the radiographs of a 7-year-old girl who was evaluated for a visible elbow
deformity by a foster parent. She thought the child fell, but her history was vague. On physical
examination, a large prominence was seen over the posterolateral elbow, and the girl lacks the terminal 20
degrees of elbow extension. She has 75 degrees of elbow pronation and supination. She was nontender
on examination. What is the most appropriate next treatment step?

1. Child abuse workup


2. Closed reduction
3. Open reduction with possible osteotomy
4. Observation

PREFERRED RESPONSE: 4

2013 American Academy of Orthopaedic Surgeons 2013 Pediatric Orthopaedic Self-Assessment Examination
104 American Academy of Orthopaedic Surgeons

DISCUSSION
The most appropriate management of this condition is observation. The patient most likely has a
congenital dislocation of the radial head, although this may also represent a posttraumatic deformity.
The absence of findings on physical examination speaks against an acute injury. The appearance of the
radial head reveals the typical findings of a congenital dislocation, namely the convex appearance of the
proximal radial articular surface. These children typically have very functional range of motion and do
not require treatment unless they are symptomatic. There is nothing in this childs history to suggest
abuse.

RECOMMENDED READINGS
Manske PR, Oberg KC. Classification and developmental biology of congenital anomalies of the hand and
upper extremity. J Bone Joint Surg Am. 2009 Jul;91 Suppl 4:3-18. PubMed PMID: 19571062.
Agnew DK, Davis RJ. Congenital unilateral dislocation of the radial head. J Pediatr Orthop. 1993 Jul-
Aug;13(4):526-8. PubMed PMID: 8370788.

Figure 114a Figure 114b

Question 114
Figures 114a and 114b are the radiographs of a 5-year-old girl who fell from monkey bars. What is the
first consideration regarding treatment for this injury?

1. Open reduction of the radiocapitellar joint


2. Reconstruction of the annular ligament
3. Closed reduction of the radial head dislocation and placement of a pin across the
radiocapitellar joint
4. Accomplishing and maintaining a reduction of the ulnar fracture

PREFERRED RESPONSE: 4

2013 American Academy of Orthopaedic Surgeons 2013 Pediatric OrthopaedicSelf-Assessment Examination


2013 Pediatric Orthopaedic Self-Assessment Examination Answer Book 105

DISCUSSION
The radiographs show an ulnar shaft fracture with a radial head dislocation (Monteggia fracture). The
ulnar shaft fracture requires reduction, either closed or open. Once the ulnar fracture has been reduced,
usually the radiocapitallar joint will relocate with closed manipulation. In patients for whom the
radiocapitallar joint does not relocate after ulnar reduction, or in patients with missed Monteggia injury,
reduction of the radiocapitellar joint usually requires opening of the elbow capsule and reconstruction of
the annular ligament.

RECOMMENDED READINGS
Nakamura K, Hirachi K, Uchiyama S, Takahara M, Minami A, Imaeda T, Kato H. Long-term clinical and
radiographic outcomes after open reduction for missed Monteggia fracture-dislocations in children. J
Bone Joint Surg Am. 2009 Jun;91(6):1394-404. PubMed PMID: 19487517.
Tan JW, Mu MZ, Liao GJ, Li JM. Pathology of the annular ligament in paediatric Monteggia fractures.
Injury. 2008 Apr;39(4):451-5. Epub 2007 Nov 19. PubMed PMID: 18005963.

Figure 115a Figure 115b

CLINICAL SITUATION FOR QUESTIONS 115 THROUGH 117


Figures 115a and 115b show the radiographs of a child who sustained a fall from his bunk bed onto an
outstretched arm.

Question 115
What is the best description of this fracture?

1. Gartland type I supracondylar humerus fracture


2. Gartland type II supracondylar humerus fracture
3. Gartland type III supracondylar humerus fracture
4. Flexion-type supracondylar humerus fracture

PREFERRED RESPONSE: 3

2013 American Academy of Orthopaedic Surgeons 2013 Pediatric Orthopaedic Self-Assessment Examination
106 American Academy of Orthopaedic Surgeons

Question 116
What is the most appropriate initial treatment?

1. Closed reduction and casting


2. Open reduction and plating
3. Skeletal traction with an olecranon pin
4. Attempted closed reduction and percutaneous pinning

PREFERRED RESPONSE: 4

Question 117
The patient was unable to extend his wrist on preoperative examination. This was caused by injury to the

1. ulnar nerve.
2. radial nerve.
3. recurrent motor branch of the median nerve.
4. anterior interosseous branch of the median nerve.

PREFERRED RESPONSE: 2

DISCUSSION FOR QUESTIONS 115 THROUGH 117


The radiographs show a Gartland type III supracondylar humerus fracture. Although multiple
classification systems exist that describe supracondylar humerus fractures, Gartlands classification
is simple and widely used. Type I fractures are nondisplaced according to this classification. Type II
fractures are displaced with a variable amount of angulation, but the posterior cortex of the humerus is
intact. Type III fractures are completely displaced with no cortical contact. In flexion-type supracondylar
humerus fractures, the distal fragments are anterior to the humeral shaft. The most appropriate treatment
method for this fracture is attempted closed reduction and pin fixation. Closed reduction and casting is
inappropriate for a Gartland type III supracondylar humerus fracture. Although open reduction may be
required in some cases, most supracondylar humerus fractures, even those that are severely displaced, can
be treated with closed reduction and percutaneous pin fixation (and this should be the initial approach).
Although olecranon traction is an option in managing these fractures, it involves repeated radiographs
and prolonged hospitalization and is not widely used in North America. The preoperative examination is
consistent with a radial nerve injury. Neuropraxias are common in supracondylar humerus fractures, with
anterior interosseous nerve injury being the most common, followed by radial nerve. Ulnar nerve injuries
occur in flexion-type supracondylar fractures and occasionally as the result of medial pin placement during
surgery. The recurrent motor branch of the median nerve supplies the thenar muscles.

2013 American Academy of Orthopaedic Surgeons 2013 Pediatric OrthopaedicSelf-Assessment Examination


2013 Pediatric Orthopaedic Self-Assessment Examination Answer Book 107

RECOMMENDED READINGS FOR QUESTIONS 115 THROUGH 117


Otsuka NY, Kasser JR. Supracondylar Fractures of the Humerus in Children. J Am Acad Orthop Surg.
1997 Jan;5(1):19-26. PubMed PMID: 10797204.
Herring JA, ed. Tachdjians Pediatric Orthopaedics. Vol 3. 4th ed. Philadelphia, PA: Saunders Elsevier;
2008: 2451-2479.
Kasser JR, Beaty JH. Supracondylar fractures of the distal humerus. In: Beaty JH, Kasser JR, eds.
Rockwood and Wilkins Fractures in Children. Philadelphia, PA: Lippincott Williams & Wilkins;
2006:544.
Babal JC, Mehlman CT, Klein G. Nerve injuries associated with pediatric supracondylar humeral
fractures: a meta-analysis. J Pediatr Orthop. 2010 Apr-May;30(3):253-63. PubMed PMID: 20357592.

END OF SERIES

Figure 118

Question 118
Figure 118 is the radiograph of a 2-year-old boy who had an isolated injury. An immediate hip spica cast
was chosen as the method of treatment. Which of the following techniques increases risk for compartment
syndrome in the left leg following cast placement?

1. Using a single-leg spica instead of a double-leg spica


2. Wrapping the left short-leg portion of the cast first, then pulling traction through the
fracture site
3. Placing the cast in an emergency department setting under sedation
4. Placing the cast within 24 hours of the injury

PREFERRED RESPONSE: 2

2013 American Academy of Orthopaedic Surgeons 2013 Pediatric Orthopaedic Self-Assessment Examination
108 American Academy of Orthopaedic Surgeons

DISCUSSION
Spica casting remains the standard of care for diaphyseal femur fractures in this age group. Spica casting
has been shown effective when the cast is placed by experienced personnel while the patient is relaxed,
whether in a setting using sedation or general anesthesia. Immediate spica casting has not been associated
with a higher rate of fracture shortening or other complications compared to traction and delayed casting.
To avoid the potential for compartment syndrome in the leg, the body and upper-leg portions of the cast
should be placed first, followed by the lower-leg portion after the initial portion of the cast has been
molded and the material has hardened.

RECOMMENDED READINGS
Infante AF Jr, Albert MC, Jennings WB, Lehner JT. Immediate hip spica casting for femur fractures in
pediatric patients. A review of 175 patients. Clin Orthop Relat Res. 2000 Jul;(376):106-12. PubMed
PMID: 10906864.
Epps HR, Molenaar E, Oconnor DP. Immediate single-leg spica cast for pediatric femoral diaphysis
fractures. J Pediatr Orthop. 2006 Jul-Aug;26(4):491-6. PubMed PMID: 16791068.
Mubarak SJ, Frick S, Sink E, Rathjen K, Noonan KJ. Volkmann contracture and compartment
syndromes after femur fractures in children treated with 90/90 spica casts. J Pediatr Orthop. 2006 Sep-
Oct;26(5):567-72. PubMed PMID: 16932092.
Mansour AA 3rd, Wilmoth JC, Mansour AS, Lovejoy SA, Mencio GA, Martus JE. Immediate spica
casting of pediatric femoral fractures in the operating room versus the emergency department: comparison
of reduction, complications, and hospital charges. J Pediatr Orthop. 2010 Dec;30(8):813-7. PubMed
PMID: 21102206.

2013 American Academy of Orthopaedic Surgeons 2013 Pediatric OrthopaedicSelf-Assessment Examination


2013 Pediatric Orthopaedic Self-Assessment Examination Answer Book 109

Figure 119a Figure 119b

Question 119
Figures 119a and 119b are the anteroposterior (AP) and lateral radiographs of a 15-year-old boy who
sustained an injury to his elbow after a fall while skateboarding. What structure is most at risk during the
surgical repair of this lesion?

1. Median nerve
2. Ulnar nerve
3. Brachial artery
4. Ulnar collateral ligament

PREFERRED RESPONSE: 2

DISCUSSION
The structure most at risk at the time of surgical repair is the ulnar nerve. The lateral radiograph shows
a bony structure in the joint space that represents the medial epicondyle, which is absent from its usual
location on the AP radiograph. The median nerve and the brachial artery should be well away from the
surgical field. The ulnar collateral ligament is often attached directly to the medial epicondyle.

RECOMMENDED READINGS
Smith BG, Pierz KA. Open reduction and internal fixation of the medial epicondyle. In: Wiesel SW,
ed. Operative Techniques in Orthopaedic Surgery. Philadelphia, PA: Lippincott Williams & Wilkins;
2009:chap 7.
Lee HH, Shen HC, Chang JH, Lee CH, Wu SS. Operative treatment of displaced medial epicondyle
fractures in children and adolescents. J Shoulder Elbow Surg. 2005 Mar-Apr;14(2):178-85. PubMed
PMID: 15789012.

2013 American Academy of Orthopaedic Surgeons 2013 Pediatric Orthopaedic Self-Assessment Examination
110 American Academy of Orthopaedic Surgeons

Question 120
A 12-year-old boy from Maine has an atraumatic knee effusion. He has no fever, mild tenderness, and
lacks 10 degrees of flexion and extension. He continues to play basketball. He had flu-like symptoms
3 months ago. Radiographs and white blood cell count (WBC) findings are normal. His erythrocyte
sedimentation rate is 53 mm/h (reference range, 0-20 mm/h) and C-reactive protein is 3.2 mg/L (reference
range, 0.08-3.1 mg/L). The knee aspiration WBC count is 55000 leukocytes/mL (a synovial fluid count
< 200 leukocytes/mL is within defined limits) with no organisms seen. What is the most appropriate initial
treatment?

1. Administer Lyme titer and oral doxycycline


2. Operative lavage of the knee and start intravenous cephalosporin
3. Start nonsteroidal anti-inflammatory drugs (NSAIDs) and refer to rheumatology
4. Hospitalization for intravenous cephalosporin

PREFERRED RESPONSE: 1

DISCUSSION
This history suggests second-stage Lyme infection, which is common in the Northeastern United States.
The condition often goes undiagnosed during the initial flu-like illness. Without treatment, Lyme disease
progresses to the arthritic stage in 3 to 6 months. The Lyme antibody test should be ordered for this
patient. Lyme is effectively treated with a 1-month course of doxycycline and does not require a surgical
lavage. This history does not suggest acute septic arthritis or osteomyelitis because the boy can bear
weight and has preserved range of motion with little pain and no fever despite his high inflammatory
markers. Inflammatory monoarticular arthritis is less likely at this age, and Lyme disease needs to be
excluded first in endemic areas. NSAIDs will not resolve Lyme disease without antibiotics.

RECOMMENDED READINGS
Smith BG, Cruz AI Jr, Milewski MD, Shapiro ED. Lyme disease and the orthopaedic implications of lyme
arthritis. J Am Acad Orthop Surg. 2011 Feb;19(2):91-100. Review. PubMed PMID: 21292932.
Milewski MD, Cruz AI Jr, Miller CP, Peterson AT, Smith BG. Lyme arthritis in children presenting with
joint effusions. J Bone Joint Surg Am. 2011 Feb 2;93(3):252-60. Erratum in: J Bone Joint Surg Am. 2011
Feb;93(3):e11. PubMed PMID: 21266639.

2013 American Academy of Orthopaedic Surgeons 2013 Pediatric OrthopaedicSelf-Assessment Examination


2013 Pediatric Orthopaedic Self-Assessment Examination Answer Book 111

Figure 121a Figure 121b

Question 121
Figures 121a and 121b are the anteroposterior (AP) and lateral radiographs of the right elbow of a 7-year-
old boy who fell off the monkey bars onto his outstretched right hand. Immediate pain and swelling were
noted around his elbow; there were no other injuries. His hand was neurovascularly intact. What is the
best treatment for this fracture?

1. Closed reduction and casting in the emergency department


2. Closed reduction and percutaneous pinning of the fracture
3. Open reduction and plate fixation of the fracture in the operating room with early mobilization
and no cast
4. Cast immobilization in the emergency department with the expectation that this injury will
heal and remodel uneventfully

PREFERRED RESPONSE: 2

DISCUSSION
Displaced supracondylar fractures are best treated with surgical closed reduction and pin fixation
followed by casting for 3 weeks. Closed reduction alone requires hyperflexion to hold the reduction and
poses higher risk for compartment syndrome and Volkmann ischemia. Plate fixation in this age group
is unnecessary considering robust periosteum and rapid healing with pin fixation. Casting the fracture
without reduction will lead to a malunion that does not usually remodel. The radiographs reveal that
the anterior humeral line does not intersect the capitellum in the lateral view, and the Baumann angle is
disrupted in the AP view.

RECOMMENDED READINGS
Supracondylar fractures of the distal humerus. In: Beaty JH, Kasser JR, eds. Rockwood and Wilkins
Fractures in Children. 6th ed. Philadelphia, PA: Lippincott Williams & Wilkins; 2006:543-590.
Omid R, Choi PD, Skaggs DL. Supracondylar humeral fractures in children. J Bone Joint Surg Am. 2008
May;90(5):1121-32. Review. PubMed PMID: 18451407.

2013 American Academy of Orthopaedic Surgeons 2013 Pediatric Orthopaedic Self-Assessment Examination
112 American Academy of Orthopaedic Surgeons

Figure 122

Question 122
Figure 122 is the radiograph of an 11-year-old gymnast who was evaluated for lateral foot pain that has
been increasing during the past 6 months. She had 3 ankle sprains during the last 6 months. She was
prescribed proprioceptive ankle exercises, orthotics, and a trial of a cast immobilization, but experienced
no relief. She has foot pain with daily activity and is now unable to participate in gymnastics. What is the
most appropriate course of action?

1. Discontinue gymnastics
2. Cast immobilization
3. Surgical treatment
4. Continued physical therapy

PREFERRED RESPONSE: 3

DISCUSSION
The radiograph shows a calcaneonavicular coalition. Surgical treatment of the coalition should result
in symptom improvement. Nonsurgical treatment has been tried and failed. She has pain with daily
activities, so discontinuing gymnastics is not likely to help. Cast treatment may provide temporary relief
but does not provide relief once the cast is removed. Continued physical therapy is not likely to help
because it has previously failed.

RECOMMENDED READINGS
Mubarak SJ, Patel PN, Upasani VV, Moor MA, Wenger DR. Calcaneonavicular coalition: treatment by
excision and fat graft. J Pediatr Orthop. 2009 Jul-Aug;29(5):418-26. PubMed PMID: 19568010.
Zaw H, Calder JD. Tarsal coalitions. Foot Ankle Clin. 2010 Jun;15(2):349-64. Review. PubMed PMID:
20534361.

2013 American Academy of Orthopaedic Surgeons 2013 Pediatric OrthopaedicSelf-Assessment Examination


Working to achieve your
health care policy goals in Washington

Since 2011, secured more than $54 million for orthopaedic research
Obtained custom device language in the House and Senate FDA User Fee bills
Helped preserve Medicare payments for imaging services

See more legislative and regulatory wins


www.aaos.org/dc

Your AAOS membership,


delivering exceptional value to you.

You might also like